Stay ahead of learning milestones! Enroll in a class over the summer!

G
Topic
First Poster
Last Poster
k a May Highlights and 2025 AoPS Online Class Information
jlacosta   0
Thursday at 11:16 PM
May is an exciting month! National MATHCOUNTS is the second week of May in Washington D.C. and our Founder, Richard Rusczyk will be presenting a seminar, Preparing Strong Math Students for College and Careers, on May 11th.

Are you interested in working towards MATHCOUNTS and don’t know where to start? We have you covered! If you have taken Prealgebra, then you are ready for MATHCOUNTS/AMC 8 Basics. Already aiming for State or National MATHCOUNTS and harder AMC 8 problems? Then our MATHCOUNTS/AMC 8 Advanced course is for you.

Summer camps are starting next month at the Virtual Campus in math and language arts that are 2 - to 4 - weeks in duration. Spaces are still available - don’t miss your chance to have an enriching summer experience. There are middle and high school competition math camps as well as Math Beasts camps that review key topics coupled with fun explorations covering areas such as graph theory (Math Beasts Camp 6), cryptography (Math Beasts Camp 7-8), and topology (Math Beasts Camp 8-9)!

Be sure to mark your calendars for the following upcoming events:
[list][*]May 9th, 4:30pm PT/7:30pm ET, Casework 2: Overwhelming Evidence — A Text Adventure, a game where participants will work together to navigate the map, solve puzzles, and win! All are welcome.
[*]May 19th, 4:30pm PT/7:30pm ET, What's Next After Beast Academy?, designed for students finishing Beast Academy and ready for Prealgebra 1.
[*]May 20th, 4:00pm PT/7:00pm ET, Mathcamp 2025 Qualifying Quiz Part 1 Math Jam, Problems 1 to 4, join the Canada/USA Mathcamp staff for this exciting Math Jam, where they discuss solutions to Problems 1 to 4 of the 2025 Mathcamp Qualifying Quiz!
[*]May 21st, 4:00pm PT/7:00pm ET, Mathcamp 2025 Qualifying Quiz Part 2 Math Jam, Problems 5 and 6, Canada/USA Mathcamp staff will discuss solutions to Problems 5 and 6 of the 2025 Mathcamp Qualifying Quiz![/list]
Our full course list for upcoming classes is below:
All classes run 7:30pm-8:45pm ET/4:30pm - 5:45pm PT unless otherwise noted.

Introductory: Grades 5-10

Prealgebra 1 Self-Paced

Prealgebra 1
Tuesday, May 13 - Aug 26
Thursday, May 29 - Sep 11
Sunday, Jun 15 - Oct 12
Monday, Jun 30 - Oct 20
Wednesday, Jul 16 - Oct 29

Prealgebra 2 Self-Paced

Prealgebra 2
Wednesday, May 7 - Aug 20
Monday, Jun 2 - Sep 22
Sunday, Jun 29 - Oct 26
Friday, Jul 25 - Nov 21

Introduction to Algebra A Self-Paced

Introduction to Algebra A
Sunday, May 11 - Sep 14 (1:00 - 2:30 pm ET/10:00 - 11:30 am PT)
Wednesday, May 14 - Aug 27
Friday, May 30 - Sep 26
Monday, Jun 2 - Sep 22
Sunday, Jun 15 - Oct 12
Thursday, Jun 26 - Oct 9
Tuesday, Jul 15 - Oct 28

Introduction to Counting & Probability Self-Paced

Introduction to Counting & Probability
Thursday, May 15 - Jul 31
Sunday, Jun 1 - Aug 24
Thursday, Jun 12 - Aug 28
Wednesday, Jul 9 - Sep 24
Sunday, Jul 27 - Oct 19

Introduction to Number Theory
Friday, May 9 - Aug 1
Wednesday, May 21 - Aug 6
Monday, Jun 9 - Aug 25
Sunday, Jun 15 - Sep 14
Tuesday, Jul 15 - Sep 30

Introduction to Algebra B Self-Paced

Introduction to Algebra B
Tuesday, May 6 - Aug 19
Wednesday, Jun 4 - Sep 17
Sunday, Jun 22 - Oct 19
Friday, Jul 18 - Nov 14

Introduction to Geometry
Sunday, May 11 - Nov 9
Tuesday, May 20 - Oct 28
Monday, Jun 16 - Dec 8
Friday, Jun 20 - Jan 9
Sunday, Jun 29 - Jan 11
Monday, Jul 14 - Jan 19

Paradoxes and Infinity
Mon, Tue, Wed, & Thurs, Jul 14 - Jul 16 (meets every day of the week!)

Intermediate: Grades 8-12

Intermediate Algebra
Sunday, Jun 1 - Nov 23
Tuesday, Jun 10 - Nov 18
Wednesday, Jun 25 - Dec 10
Sunday, Jul 13 - Jan 18
Thursday, Jul 24 - Jan 22

Intermediate Counting & Probability
Wednesday, May 21 - Sep 17
Sunday, Jun 22 - Nov 2

Intermediate Number Theory
Sunday, Jun 1 - Aug 24
Wednesday, Jun 18 - Sep 3

Precalculus
Friday, May 16 - Oct 24
Sunday, Jun 1 - Nov 9
Monday, Jun 30 - Dec 8

Advanced: Grades 9-12

Olympiad Geometry
Tuesday, Jun 10 - Aug 26

Calculus
Tuesday, May 27 - Nov 11
Wednesday, Jun 25 - Dec 17

Group Theory
Thursday, Jun 12 - Sep 11

Contest Preparation: Grades 6-12

MATHCOUNTS/AMC 8 Basics
Friday, May 23 - Aug 15
Monday, Jun 2 - Aug 18
Thursday, Jun 12 - Aug 28
Sunday, Jun 22 - Sep 21
Tues & Thurs, Jul 8 - Aug 14 (meets twice a week!)

MATHCOUNTS/AMC 8 Advanced
Sunday, May 11 - Aug 10
Tuesday, May 27 - Aug 12
Wednesday, Jun 11 - Aug 27
Sunday, Jun 22 - Sep 21
Tues & Thurs, Jul 8 - Aug 14 (meets twice a week!)

AMC 10 Problem Series
Friday, May 9 - Aug 1
Sunday, Jun 1 - Aug 24
Thursday, Jun 12 - Aug 28
Tuesday, Jun 17 - Sep 2
Sunday, Jun 22 - Sep 21 (1:00 - 2:30 pm ET/10:00 - 11:30 am PT)
Monday, Jun 23 - Sep 15
Tues & Thurs, Jul 8 - Aug 14 (meets twice a week!)

AMC 10 Final Fives
Sunday, May 11 - Jun 8
Tuesday, May 27 - Jun 17
Monday, Jun 30 - Jul 21

AMC 12 Problem Series
Tuesday, May 27 - Aug 12
Thursday, Jun 12 - Aug 28
Sunday, Jun 22 - Sep 21
Wednesday, Aug 6 - Oct 22

AMC 12 Final Fives
Sunday, May 18 - Jun 15

AIME Problem Series A
Thursday, May 22 - Jul 31

AIME Problem Series B
Sunday, Jun 22 - Sep 21

F=ma Problem Series
Wednesday, Jun 11 - Aug 27

WOOT Programs
Visit the pages linked for full schedule details for each of these programs!


MathWOOT Level 1
MathWOOT Level 2
ChemWOOT
CodeWOOT
PhysicsWOOT

Programming

Introduction to Programming with Python
Thursday, May 22 - Aug 7
Sunday, Jun 15 - Sep 14 (1:00 - 2:30 pm ET/10:00 - 11:30 am PT)
Tuesday, Jun 17 - Sep 2
Monday, Jun 30 - Sep 22

Intermediate Programming with Python
Sunday, Jun 1 - Aug 24
Monday, Jun 30 - Sep 22

USACO Bronze Problem Series
Tuesday, May 13 - Jul 29
Sunday, Jun 22 - Sep 1

Physics

Introduction to Physics
Wednesday, May 21 - Aug 6
Sunday, Jun 15 - Sep 14
Monday, Jun 23 - Sep 15

Physics 1: Mechanics
Thursday, May 22 - Oct 30
Monday, Jun 23 - Dec 15

Relativity
Mon, Tue, Wed & Thurs, Jun 23 - Jun 26 (meets every day of the week!)
0 replies
jlacosta
Thursday at 11:16 PM
0 replies
k i Adding contests to the Contest Collections
dcouchman   1
N Apr 5, 2023 by v_Enhance
Want to help AoPS remain a valuable Olympiad resource? Help us add contests to AoPS's Contest Collections.

Find instructions and a list of contests to add here: https://artofproblemsolving.com/community/c40244h1064480_contests_to_add
1 reply
dcouchman
Sep 9, 2019
v_Enhance
Apr 5, 2023
k i Zero tolerance
ZetaX   49
N May 4, 2019 by NoDealsHere
Source: Use your common sense! (enough is enough)
Some users don't want to learn, some other simply ignore advises.
But please follow the following guideline:


To make it short: ALWAYS USE YOUR COMMON SENSE IF POSTING!
If you don't have common sense, don't post.


More specifically:

For new threads:


a) Good, meaningful title:
The title has to say what the problem is about in best way possible.
If that title occured already, it's definitely bad. And contest names aren't good either.
That's in fact a requirement for being able to search old problems.

Examples:
Bad titles:
- "Hard"/"Medium"/"Easy" (if you find it so cool how hard/easy it is, tell it in the post and use a title that tells us the problem)
- "Number Theory" (hey guy, guess why this forum's named that way¿ and is it the only such problem on earth¿)
- "Fibonacci" (there are millions of Fibonacci problems out there, all posted and named the same...)
- "Chinese TST 2003" (does this say anything about the problem¿)
Good titles:
- "On divisors of a³+2b³+4c³-6abc"
- "Number of solutions to x²+y²=6z²"
- "Fibonacci numbers are never squares"


b) Use search function:
Before posting a "new" problem spend at least two, better five, minutes to look if this problem was posted before. If it was, don't repost it. If you have anything important to say on topic, post it in one of the older threads.
If the thread is locked cause of this, use search function.

Update (by Amir Hossein). The best way to search for two keywords in AoPS is to input
[code]+"first keyword" +"second keyword"[/code]
so that any post containing both strings "first word" and "second form".


c) Good problem statement:
Some recent really bad post was:
[quote]$lim_{n\to 1}^{+\infty}\frac{1}{n}-lnn$[/quote]
It contains no question and no answer.
If you do this, too, you are on the best way to get your thread deleted. Write everything clearly, define where your variables come from (and define the "natural" numbers if used). Additionally read your post at least twice before submitting. After you sent it, read it again and use the Edit-Button if necessary to correct errors.


For answers to already existing threads:


d) Of any interest and with content:
Don't post things that are more trivial than completely obvious. For example, if the question is to solve $x^{3}+y^{3}=z^{3}$, do not answer with "$x=y=z=0$ is a solution" only. Either you post any kind of proof or at least something unexpected (like "$x=1337, y=481, z=42$ is the smallest solution). Someone that does not see that $x=y=z=0$ is a solution of the above without your post is completely wrong here, this is an IMO-level forum.
Similar, posting "I have solved this problem" but not posting anything else is not welcome; it even looks that you just want to show off what a genius you are.

e) Well written and checked answers:
Like c) for new threads, check your solutions at least twice for mistakes. And after sending, read it again and use the Edit-Button if necessary to correct errors.



To repeat it: ALWAYS USE YOUR COMMON SENSE IF POSTING!


Everything definitely out of range of common sense will be locked or deleted (exept for new users having less than about 42 posts, they are newbies and need/get some time to learn).

The above rules will be applied from next monday (5. march of 2007).
Feel free to discuss on this here.
49 replies
ZetaX
Feb 27, 2007
NoDealsHere
May 4, 2019
3 right-angled triangle area
NicoN9   1
N 3 minutes ago by Mathzeus1024
Source: Japan Junior MO Preliminary 2020 P1
Right angled triangle $ABC$, and a square are drawn as shown below. Three numbers written below implies each of the area of shaded small right angled triangle. Find the value of $AB/AC$.

IMAGE
1 reply
NicoN9
Yesterday at 6:08 AM
Mathzeus1024
3 minutes ago
IMO Genre Predictions
ohiorizzler1434   12
N 4 minutes ago by NO_SQUARES
Everybody, with IMO upcoming, what are you predictions for the problem genres?


Personally I predict: predict
12 replies
ohiorizzler1434
6 hours ago
NO_SQUARES
4 minutes ago
two sequences of positive integers and inequalities
rmtf1111   50
N 4 minutes ago by math-olympiad-clown
Source: EGMO 2019 P5
Let $n\ge 2$ be an integer, and let $a_1, a_2, \cdots , a_n$ be positive integers. Show that there exist positive integers $b_1, b_2, \cdots, b_n$ satisfying the following three conditions:

$\text{(A)} \ a_i\le b_i$ for $i=1, 2, \cdots , n;$

$\text{(B)} \ $ the remainders of $b_1, b_2, \cdots, b_n$ on division by $n$ are pairwise different; and

$\text{(C)} \ $ $b_1+b_2+\cdots b_n \le n\left(\frac{n-1}{2}+\left\lfloor \frac{a_1+a_2+\cdots a_n}{n}\right \rfloor \right)$

(Here, $\lfloor x \rfloor$ denotes the integer part of real number $x$, that is, the largest integer that does not exceed $x$.)
50 replies
rmtf1111
Apr 10, 2019
math-olympiad-clown
4 minutes ago
inequalities
Tamako22   0
5 minutes ago
let $a,b,c> 1,\dfrac{1}{1+a}+\dfrac{1}{1+b}+\dfrac{1}{1+c}=1.$
prove that$$\sqrt{a}+\sqrt{b}+\sqrt{c}\ge \dfrac{2}{\sqrt{a}}+\dfrac{2}{\sqrt{b}}+\dfrac{2}{\sqrt{c}}$$
0 replies
Tamako22
5 minutes ago
0 replies
Problem 6
SlovEcience   2
N 8 minutes ago by mashumaro
Given two points A and B on the unit circle. The tangents to the circle at A and B intersect at point P. Then:
\[ p = \frac{2ab}{a + b} \], \[ p, a, b \in \mathbb{C} \]
2 replies
SlovEcience
3 hours ago
mashumaro
8 minutes ago
A coincidence about triangles with common incenter
flower417477   3
N 21 minutes ago by mashumaro
$\triangle ABC,\triangle ADE$ have the same incenter $I$.Prove that $BCDE$ is concyclic iff $BC,DE,AI$ is concurrent
3 replies
flower417477
Apr 30, 2025
mashumaro
21 minutes ago
this geo is scarier than the omega variant
AwesomeYRY   11
N 28 minutes ago by LuminousWolverine
Source: TSTST 2021/6
Triangles $ABC$ and $DEF$ share circumcircle $\Omega$ and incircle $\omega$ so that points $A,F,B,D,C,$ and $E$ occur in this order along $\Omega$. Let $\Delta_A$ be the triangle formed by lines $AB,AC,$ and $EF,$ and define triangles $\Delta_B, \Delta_C, \ldots, \Delta_F$ similarly. Furthermore, let $\Omega_A$ and $\omega_A$ be the circumcircle and incircle of triangle $\Delta_A$, respectively, and define circles $\Omega_B, \omega_B, \ldots, \Omega_F, \omega_F$ similarly.

(a) Prove that the two common external tangents to circles $\Omega_A$ and $\Omega_D$ and the two common external tangents to $\omega_A$ and $\omega_D$ are either concurrent or pairwise parallel.

(b) Suppose that these four lines meet at point $T_A$, and define points $T_B$ and $T_C$ similarly. Prove that points $T_A,T_B$, and $T_C$ are collinear.

Nikolai Beluhov
11 replies
AwesomeYRY
Dec 13, 2021
LuminousWolverine
28 minutes ago
No function f on reals such that f(f(x))=x^2-2
N.T.TUAN   17
N 38 minutes ago by Assassino9931
Source: VietNam TST 1990
Prove that there does not exist a function $f: \mathbb R\to\mathbb R$ such that $f(f(x))=x^2-2$ for all $x\in\mathbb R$.
17 replies
N.T.TUAN
Dec 31, 2006
Assassino9931
38 minutes ago
Hard diophant equation
MuradSafarli   4
N 41 minutes ago by iniffur
Find all positive integers $x, y, z, t$ such that the equation

$$
2017^x + 6^y + 2^z = 2025^t
$$
is satisfied.
4 replies
MuradSafarli
Yesterday at 6:12 PM
iniffur
41 minutes ago
Geometry that "looks" hard
Pmshw   3
N an hour ago by Lemmas
Source: Iran 2nd round 2022 P6
we have an isogonal triangle $ABC$ such that $BC=AB$. take a random $P$ on the altitude from $B$ to $AC$.
The circle $(ABP)$ intersects $AC$ second time in $M$. Take $N$ such that it's on the segment $AC$ and $AM=NC$ and $M \neq N$.The second intersection of $NP$ and circle $(APB)$ is $X$ , ($X \neq P$) and the second intersection of $AB$ and circle $(APN)$ is $Y$ ,($Y \neq A$).The tangent from $A$ to the circle $(APN)$ intersects the altitude from $B$ at $Z$.
Prove that $CZ$ is tangent to circle $(PXY)$.
3 replies
1 viewing
Pmshw
May 9, 2022
Lemmas
an hour ago
inequalities
Cobedangiu   2
N an hour ago by Cobedangiu
$a,b,c>0$ and $\sum ab=\dfrac{1}{3}$. Prove that:
$\sum \dfrac{1}{a^2-bc+1}\le 3$
2 replies
Cobedangiu
Today at 4:06 AM
Cobedangiu
an hour ago
IMO Shortlist Problems
ABCD1728   6
N an hour ago by ABCD1728
Source: IMO official website
Where can I get the official solution for ISL before 2005? The official website only has solutions after 2006. Thanks :)
6 replies
ABCD1728
Yesterday at 12:44 PM
ABCD1728
an hour ago
Find the product
sqing   1
N 2 hours ago by Primeniyazidayi
Source: Ecrin_eren
The roots of $ x^3 - 2x^2 - 11x + k=0 $ are $r_1, r_2,  r_3 $ and $ r_1+2 r_2+3 r_3= 0.$ Find the product of all possible values of $ k .$
1 reply
sqing
2 hours ago
Primeniyazidayi
2 hours ago
Some free permutation
IndoMathXdZ   23
N 2 hours ago by Jupiterballs
Source: ISL 2020 N7
Let $\mathcal{S}$ be a set consisting of $n \ge 3$ positive integers, none of which is a sum of two other distinct members of $\mathcal{S}$. Prove that the elements of $\mathcal{S}$ may be ordered as $a_1, a_2, \dots, a_n$ so that $a_i$ does not divide $a_{i - 1} + a_{i + 1}$ for all $i = 2, 3, \dots, n - 1$.
23 replies
IndoMathXdZ
Jul 20, 2021
Jupiterballs
2 hours ago
Excircle Tangency Points Concyclic with A
tastymath75025   35
N Apr 24, 2025 by bin_sherlo
Source: USA Winter TST for IMO 2019, Problem 6, by Ankan Bhattacharya
Let $ABC$ be a triangle with incenter $I$, and let $D$ be a point on line $BC$ satisfying $\angle AID=90^{\circ}$. Let the excircle of triangle $ABC$ opposite the vertex $A$ be tangent to $\overline{BC}$ at $A_1$. Define points $B_1$ on $\overline{CA}$ and $C_1$ on $\overline{AB}$ analogously, using the excircles opposite $B$ and $C$, respectively.

Prove that if quadrilateral $AB_1A_1C_1$ is cyclic, then $\overline{AD}$ is tangent to the circumcircle of $\triangle DB_1C_1$.

Ankan Bhattacharya
35 replies
tastymath75025
Jan 21, 2019
bin_sherlo
Apr 24, 2025
Excircle Tangency Points Concyclic with A
G H J
G H BBookmark kLocked kLocked NReply
Source: USA Winter TST for IMO 2019, Problem 6, by Ankan Bhattacharya
The post below has been deleted. Click to close.
This post has been deleted. Click here to see post.
tastymath75025
3223 posts
#1 • 6 Y
Y by anantmudgal09, nguyendangkhoa17112003, megarnie, Adventure10, Mango247, Rounak_iitr
Let $ABC$ be a triangle with incenter $I$, and let $D$ be a point on line $BC$ satisfying $\angle AID=90^{\circ}$. Let the excircle of triangle $ABC$ opposite the vertex $A$ be tangent to $\overline{BC}$ at $A_1$. Define points $B_1$ on $\overline{CA}$ and $C_1$ on $\overline{AB}$ analogously, using the excircles opposite $B$ and $C$, respectively.

Prove that if quadrilateral $AB_1A_1C_1$ is cyclic, then $\overline{AD}$ is tangent to the circumcircle of $\triangle DB_1C_1$.

Ankan Bhattacharya
This post has been edited 1 time. Last edited by tastymath75025, Jan 21, 2019, 5:08 PM
Z K Y
The post below has been deleted. Click to close.
This post has been deleted. Click here to see post.
math_pi_rate
1218 posts
#2 • 21 Y
Y by mathman3880, Generic_Username, nguyendangkhoa17112003, AlastorMoody, anantmudgal09, brokendiamond, amar_04, Wizard_32, k12byda5h, srijonrick, KST2003, MathsLion, hakN, mijail, megarnie, crazyeyemoody907, Aryan-23, Adventure10, Kingsbane2139, Ywgh1, Funcshun840
Nice problem (although easy for a USA TST P6):

[asy]
 /* Geogebra to Asymptote conversion, documentation at artofproblemsolving.com/Wiki go to User:Azjps/geogebra */
import graph; size(15cm); 
real labelscalefactor = 0.5; /* changes label-to-point distance */
pen dps = linewidth(0.7) + fontsize(10); defaultpen(dps); /* default pen style */ 
pen dotstyle = black; /* point style */ 
real xmin = -14.75, xmax = 11.45, ymin = -3.79, ymax = 7.45;  /* image dimensions */
pen wrwrwr = rgb(0.3803921568627451,0.3803921568627451,0.3803921568627451); pen wwzzff = rgb(0.4,0.6,1); pen ffxfqq = rgb(1,0.4980392156862745,0); pen qqccqq = rgb(0,0.8,0); 
 /* draw figures */
draw((-4.781523029254206,2.683377195388904)--(6.871149068786946,5.218374928896586), linewidth(0.4) + wrwrwr); 
draw(circle((0.7015597483382036,1.4355753715441475), 2.546602274766395), linewidth(0.4) + blue); 
draw((-4.781523029254206,2.683377195388904)--(2.0614153440904808,-0.7175562789934126), linewidth(0.4) + wrwrwr); 
draw((2.0614153440904808,-0.7175562789934126)--(6.871149068786946,5.218374928896586), linewidth(0.4) + wrwrwr); 
draw((-0.7268440288168109,-0.6727049217884695)--(-0.66,3.58), linewidth(0.4) + wrwrwr); 
draw((-7.217533164119876,-0.5682970536602701)--(-2.76,-0.64), linewidth(0.4) + wrwrwr); 
draw((-7.217533164119876,-0.5682970536602701)--(-0.09465334849420755,0.9812603385617286), linewidth(0.4) + wrwrwr); 
draw(circle((-0.9294856026840693,-10.516443266922067), 11.768821318012856), linewidth(0.4) + linetype("2 2") + wwzzff); 
draw((-7.217533164119876,-0.5682970536602701)--(-0.66,3.58), linewidth(0.4) + wrwrwr); 
draw(circle((-3.943883513499975,1.5047383027747945), 3.8846624103269236), linewidth(0.4) + ffxfqq); 
draw((-0.66,3.58)--(-2.76,-0.64), linewidth(0.4) + qqccqq); 
draw((-2.76,-0.64)--(4.7,-0.76), linewidth(0.4) + qqccqq); 
draw((4.7,-0.76)--(-0.66,3.58), linewidth(0.4) + qqccqq); 
draw((-3.9387665820599382,1.505851473169865)--(3.0875794770264293,0.545579304049496), linewidth(0.4) + wrwrwr); 
 /* dots and labels */
dot((-0.66,3.58),dotstyle); 
label("$A$", (-0.69,3.87), NE * labelscalefactor); 
dot((-2.76,-0.64),dotstyle); 
label("$B$", (-3.15,-1.15), NE * labelscalefactor); 
dot((4.7,-0.76),dotstyle); 
label("$C$", (4.93,-1.19), NE * labelscalefactor); 
dot((-0.09465334849420755,0.9812603385617286),linewidth(4pt) + dotstyle); 
label("$I$", (0.15,0.57), NE * labelscalefactor); 
dot((6.871149068786946,5.218374928896586),linewidth(4pt) + dotstyle); 
label("$I_B$", (7.13,5.43), NE * labelscalefactor); 
dot((-4.781523029254206,2.683377195388904),linewidth(4pt) + dotstyle); 
label("$I_C$", (-5.33,2.89), NE * labelscalefactor); 
dot((-1.8356826708257379,1.2174376805311355),linewidth(4pt) + dotstyle); 
label("$C_1$", (-2.35,1.71), NE * labelscalefactor); 
dot((3.0875794770264293,0.545579304049496),linewidth(4pt) + dotstyle); 
label("$B_1$", (3.09,-0.03), NE * labelscalefactor); 
dot((2.0614153440904808,-0.7175562789934126),linewidth(4pt) + dotstyle); 
label("$A_1$", (2.21,-1.05), NE * labelscalefactor); 
dot((-0.7268440288168109,-0.6727049217884695),linewidth(4pt) + dotstyle); 
label("$X$", (-0.71,-1.25), NE * labelscalefactor); 
dot((-7.217533164119876,-0.5682970536602701),linewidth(4pt) + dotstyle); 
label("$D$", (-7.99,-0.45), NE * labelscalefactor); 
dot((-3.9387665820599382,1.505851473169865),linewidth(4pt) + dotstyle); 
label("$M$", (-4.53,1.51), NE * labelscalefactor); 
clip((xmin,ymin)--(xmin,ymax)--(xmax,ymax)--(xmax,ymin)--cycle); 
 /* end of picture */
[/asy]

From USA TST for EGMO 2019, Problem 5, we get that $I$ lies on $B_1C_1$, and that $\odot (AB_1A_1C_1) \cap BC=X$ is the foot of the $A$-altitude. Let $I_BI_C \cap BC=T$, where $I_B$ and $I_C$ are the $B$ and $C$-excenters. Then, from that same solution, we have that $I_B,B_1,A_1$ and $I_C,C_1,A_1$ are collinear. Thus, $$-1=(T,A;I_B,I_C) \overset{A_1}{=} (X,A;B_1,C_1) \Rightarrow AB_1XC_1 \text{ is harmonic.}$$Suppose the tangents to $\odot (AB_1A_1C_1)$ at $A$ and $X$ meet on $B_1C_1$ at a point $M$. Then, as $I$ is the foot of the $A$-internal angle bisector of $\angle B_1AC_1$, so we get that $\odot (AIX)$ is the $A$-Apollonius circle of $\triangle AB_1C_1$. Also, as $\angle DIA=\angle DXA=90^{\circ}$, so we have that $D$ also lies on this circle centered at $M$, i.e. $MD^2=MA^2=MB_1 \cdot MC_1$. Hence, done. $\blacksquare$
This post has been edited 2 times. Last edited by math_pi_rate, Jan 21, 2019, 6:57 PM
Reason: Added diagram
Z K Y
The post below has been deleted. Click to close.
This post has been deleted. Click here to see post.
62861
3564 posts
#3 • 3 Y
Y by aopsuser305, Adventure10, Mango247
math_pi_rate wrote:
easy for a USA TST P6

I agree with you.

What did you all think of the problem? When I originally came up with it I thought it was genius, but now I think it's really ugly and not any fun to try and solve. Many others mentioned they thought the problem was nice, so perhaps it's just my aversion to configuration-based geometry at work here...
This post has been edited 1 time. Last edited by 62861, Jan 21, 2019, 7:14 PM
Z K Y
The post below has been deleted. Click to close.
This post has been deleted. Click here to see post.
math_pi_rate
1218 posts
#4 • 4 Y
Y by 62861, Adventure10, Mango247, ehuseyinyigit
@above I believe that the configuration is really nice and rich, and problem is also fine. However, considering at TST level, the problem could have been more difficult. Cause after finding that $A_1$ is the Bevan point of $\triangle ABC$ (again not that difficult), the above solution is quite easy to find (Probably saying this because I personally found P1 harder than this :P).
Z K Y
The post below has been deleted. Click to close.
This post has been deleted. Click here to see post.
enhanced
515 posts
#5 • 2 Y
Y by Gaussian_cyber, Adventure10
$\qquad \qquad$
This post has been edited 4 times. Last edited by enhanced, Aug 20, 2020, 6:03 AM
Z K Y
The post below has been deleted. Click to close.
This post has been deleted. Click here to see post.
Pathological
578 posts
#6 • 3 Y
Y by JasperL, yayups, Adventure10
Here is a sketch of a relatively straightforward solution which doesn't require much thinking after guessing that $A_1$ should be the Bevan point of $\triangle ABC$.


Bashy Solution
This post has been edited 1 time. Last edited by Pathological, Jan 21, 2019, 7:34 PM
Z K Y
The post below has been deleted. Click to close.
This post has been deleted. Click here to see post.
math_pi_rate
1218 posts
#7 • 2 Y
Y by Adventure10, Mango247
Btw here's another nice property of the given configuration: Let $\omega$ be the circumcircle of $\triangle AC_1D$. Suppose the tangents to $\omega$ at $A$ and $D$ meet at $K$, and let $AB_1 \cap \omega=X, DB_1 \cap \omega=Y,B_1K \cap AD=Z$. Then $MXYZ$ is cyclic, where $M$ is the midpoint of $AD$.
Z K Y
The post below has been deleted. Click to close.
This post has been deleted. Click here to see post.
GGPiku
402 posts
#8 • 1 Y
Y by Adventure10
Well, my first drawing confused me, so I will consider $D$ to be be a point on the tangent at $A$ wrt $(AC_1B_1)$ satisfying $\angle AID=90$.
Good drawing really helps you see the main steps of the problem( I did 3 sketches with some obtuse triangles to see that $AA_1$ was a diameter, or that $HI$ was the angle bisector of $C_1HB_1$ which was implying the harmonic quad).
Let $A_2,B_2,C_2$ be the excenters, let $H$ be the foot of the altitude from $A$. Since $ABC$ is the orthic triangle of $A_2B_2C_2$, by Cevian Nests we have $A_2A_1,B_2B_1,C_2C_1$ concurrent in $X$. As both $X$ and $A_1$ lie on $A_2A_1$ and $AB_1XC_1A_1$ is cyclic with diameter $AX$, if $X,A$ differ, then $AA_1\perp AX$ which implies $AA_1\parallel BC$, which is impossible. So $X=A_1$, so $AC_1HA_1B_1$ is cyclic.By Pappus, $I$ is on $C_1B_1$. Note that since $ABC$ is the orthic triangle, it follows that $A_1(C_1,B_1,H,A)=-1$, so $AB_1HC_1$ is harmonic. It follows that $\angle AHI=\angle BHI -\angle BHA=90-\frac{A}{2}-\angle AB_1C_1=90-\angle C_1AI-\angle DAC_1=\angle ADI$, so $DHIA$ is cyclic, implying that $\angle DHA=90$ and $D$ being on $BC$. Let $M$ be $AD\cap C_1B_1$. By the properties of harmonic quadrilaterals, it follows that $MH$ will be also tangent, so $MH=MA=MD$ since $\angle DHA=90$. We can easily finish by power of point from here. Another fun observation is that $B_2C_2$ meets $XI$ on the circle.
This post has been edited 6 times. Last edited by GGPiku, Jan 22, 2019, 8:50 AM
Z K Y
The post below has been deleted. Click to close.
This post has been deleted. Click here to see post.
stroller
894 posts
#10 • 2 Y
Y by Adventure10, Mango247
Anyone bashed this successfully without guessing any geometrical claims? I got a degree 7 equation to verify subject to a deg 3 one and was too tired to verify it...
Z K Y
The post below has been deleted. Click to close.
This post has been deleted. Click here to see post.
RC.
439 posts
#11 • 2 Y
Y by AlastorMoody, Adventure10
Solution avoids using \(A_1\) to be the bevan (because I couldn't find that).

\(\overline{B_1,I,C_1-}\)
Proof- Let \(M_B, M_C\) denote the midpoints of small \(\overarc{AC}, \overarc{AB}\) resp. Let \(M_BB_1 \cap \odot ABC = T \Rightarrow \Delta ATC\sim \Delta A_1BC_1 \Rightarrow \angle CAT = \angle BA_1C_1\) Similarly, let \(M_CC_1 \cap \odot ABC = T' \Rightarrow \angle BAT' = \angle CA_1B_1 \therefore \angle CAT + \angle BAT' = \angle A \Rightarrow T= T'.\) Now applying Pascal's on cyclic hexagon \((ABM_BTM_C)\) we get \(\overline{B_1,I,C_1}\) \(\quad \square\)
\(AD\) is tangent to \(\odot AC_1B_1-\)
Proo - In any \(\triangle\) let \(E,F\) be the incircle touch points on \(AC, AB\) resp. then \(AD, \odot AEF, \odot ABC\) are concurrent. Thus the result is obvious for any arbitrary \(\Delta \quad\square\)
Let \(B_1C_1 \cap AD= M\). Since \(AI\) is the angle bisector and \(AM\) the tangent we have \(MA= MI = MD\) but \(\angle MAC_1 = \angle MB_1A \Rightarrow C_1\) is the Humpty point of \(\Delta AB_1D \Rightarrow AD\) is tangent to \(\odot DB_1C_1. \quad\blacksquare\)
This post has been edited 1 time. Last edited by RC., Jan 27, 2019, 2:34 AM
Z K Y
The post below has been deleted. Click to close.
This post has been deleted. Click here to see post.
v_Enhance
6877 posts
#12 • 12 Y
Y by anantmudgal09, nguyendangkhoa17112003, RAMUGAUSS, AlastorMoody, Kanep, Aryan-23, v4913, HamstPan38825, sabkx, Adventure10, Rounak_iitr, Funcshun840
Official solutions post (also at http://web.evanchen.cc/problems.html):

We present two solutions.

First solution using spiral similarity (Ankan Bhattacharya) First, we prove the part of the problem which does not depend on the condition $A B_1 A_1 C_1$ is cyclic.

Lemma: Let $ABC$ be a triangle and define $I$, $D$, $B_1$, $C_1$ as in the problem. Moreover, let $M$ denote the midpoint of $\overline{AD}$. Then $\overline{AD}$ is tangent to $(AB_1C_1)$, and moreover $\overline{B_1 C_1} \parallel \overline{IM}$.

Proof. Let $E$ and $F$ be the tangency points of the incircle. Denote by $Z$ the Miquel point of $BFEC$, i.e.\ the second intersection of the circle with diameter $\overline{AI}$ and the circumcircle.

Note that $A$, $Z$, $D$ are collinear, by radical axis on $(ABC)$, $(AFIE)$, $(BIC)$.



[asy] 	size(11cm); 	pair A = dir(130); 	pair B = dir(210); 	pair C = dir(330); 	pair I = incenter(A, B, C); 	pair E = foot(I, C, A); 	pair B_1 = A+C-E; 	pair F = foot(I, A, B); 	pair C_1 = A+B-F; 	pair T = foot(I, B, C); 	pair Z = foot(A, I, foot(T, E, F));

filldraw(A--B--C--cycle, invisible, blue); 	draw(circumcircle(A, E, F), orange); 	pair D = extension(A, Z, B, C); 	draw(arc(dir(270), C, B), orange); 	draw(D--B, blue);

filldraw(A--B_1--C_1--cycle, invisible, blue); 	pair M = midpoint(A--D); 	filldraw(A--D--I--cycle, invisible, red); 	draw(I--M, red); 	filldraw(Z--E--F--cycle, invisible, deepgreen); 	filldraw(unitcircle, invisible, blue); 	draw(incircle(A, B, C), dotted+blue); 	draw(circumcircle(A, B_1, C_1), dashed+red);

dot("$A$", A, dir(A)); 	dot("$B$", B, dir(225)); 	dot("$C$", C, dir(315)); 	dot("$I$", I, dir(270)); 	dot("$E$", E, dir(30)); 	dot("$B_1$", B_1, dir(B_1)); 	dot("$F$", F, dir(F)); 	dot("$C_1$", C_1, dir(C_1)); 	dot("$Z$", Z, dir(Z)); 	dot("$D$", D, dir(D)); 	dot("$M$", M, dir(M));

/* TSQ Source:

!size(11cm); 	A = dir 130 	B = dir 210 R225 	C = dir 330 R315 	I = incenter A B C R270 	E = foot I C A R30 	B_1 = A+C-E 	F = foot I A B 	C_1 = A+B-F 	T := foot I B C 	Z = foot A I foot T E F

A--B--C--cycle 0.1 lightcyan / blue 	circumcircle A E F orange 	D = extension A Z B C 	!draw(arc(dir(270), C, B), orange); 	D--B blue

A--B_1--C_1--cycle 0.1 deepgreen / blue 	M = midpoint A--D 	A--D--I--cycle 0.1 lightred / red 	I--M red 	Z--E--F--cycle 0.1 deepgreen / deepgreen 	unitcircle 0.1 lightcyan / blue 	incircle A B C dotted blue 	circumcircle A B_1 C_1 dashed red

*/ 	[/asy]

Then the spiral similarity gives us \[ \frac{ZF}{ZE} = \frac{BF}{CE} = \frac{AC_1}{AB_1} \]which together with $\measuredangle FZE = \measuredangle FAE = \measuredangle BAC$ implies that $\triangle ZFE$ and $\triangle AC_1B_1$ are (directly) similar. (See IMO Shortlist 2006 G9 for a similar application of spiral similarity.)

Now the remainder of the proof is just angle chasing. First, since \[ \measuredangle DAC_1 = \measuredangle ZAF = \measuredangle ZEF = \measuredangle AB_1C_1 \]we have $\overline{AD}$ is tangent to $(AB_1C_1)$. Moreover, to see that $\overline{IM} \parallel \overline{B_1C_1}$, write \begin{align*} 		\measuredangle (\overline{AI}, \overline{B_1C_1}) 		&= \measuredangle IAC + \measuredangle AB_1C_1 = \measuredangle BAI + \measuredangle ZEF 		= \measuredangle FAI + \measuredangle ZAF \\ 		&= \measuredangle ZAI = \measuredangle MAI = \measuredangle AIM 	\end{align*}the last step since $\triangle AID$ is right with hypotenuse $\overline{AD}$, and median $\overline{IM}$. $\blacksquare$



Now we return to the present problem with the additional condition.



[asy] unitsize(100); pair A, B, C, I, A1, B1, C1, D, E, F, J, K, Z, M; B = dir(193); C = reflect((0, 1), (0, -1)) * B; I = intersectionpoints(circle(dir(270), abs(dir(270)-B)), -B--(-C))[1]; A = 2 * foot(origin, I, dir(270)) - dir(270); D = extension(B, C, I, rotate(90, I) * A); E = foot(I, C, A); F = foot(I, A, B); A1 = B + C - foot(I, B, C); B1 = C + A - E; C1 = A + B - F; M = (A + D)/2; Z = foot(I, A, D); K = 2 * foot(circumcenter(A, B1, C1), A, I) - A;

draw(unitcircle); filldraw(circumcircle(A, B1, C1), invisible, red); filldraw(circumcircle(A, E, F), invisible, heavycyan+dashed); draw(incircle(A, B, C), heavycyan+dotted); filldraw(circumcircle(A, B, C), invisible, cyan); filldraw(circumcircle(B, I, C), invisible, heavycyan+dashed); draw(arc(circumcenter(D, B1, C1), B1, D), heavyred); draw(A--D, heavyred); draw(A--B--C--cycle, heavymagenta); draw(D--B, lightmagenta); draw(A--I--D, heavycyan); draw(I--K, heavyred); draw(M--B1, dashed+heavyred);

dot("$A$", A, dir(A)); dot("$B$", B, dir(B)); dot("$C$", C, dir(10)); dot("$I$", I, dir(-I)); dot("$A_1 = V$", A1, dir(A1)); dot("$B_1$", B1, dir(10)); dot("$C_1$", C1, dir(130)); dot("$D$", D, dir(D)); dot("$E$", E, dir(40)); dot("$F$", F, dir(170)); dot("$M$", M, dir(130)); dot("$Z$", Z, dir(Z)); // dot("$K$", K, dir(K-circumcenter(A, B1, C1)));

clip(box((-3, -0.4), (1.2, 1.25))); [/asy]



Claim: Given the condition, we actually have $\angle AB_1A_1 = \angle AC_1A_1 = 90^{\circ}$.

Proof. Let $I_A$, $I_B$ and $I_C$ be the excenters of $\triangle ABC$. Then the perpendiculars to $\overline{BC}$, $\overline{CA}$, $\overline{AB}$ from $A_1$, $B_1$, $C_1$ respectively meet at the so-called Bevan point $V$ (which is the circumcenter of $\triangle I_A I_B I_C$).

Now $\triangle AB_1C_1$ has circumdiameter $\overline{AV}$. We are given $A_1$ lies on this circle, so if $V \ne A_1$ then $\overline{AA_1} \perp \overline{A_1V}$. But $\overline{A_1V} \perp \overline{BC}$ by definition, which would imply $\overline{AA_1} \parallel \overline{BC}$, which is absurd. $\blacksquare$



Claim: Given the condition the points $B_1$, $I$, $C_1$ are collinear (hence with $M$).

Proof. By Pappus theorem on $\overline{I_B A I_C}$ and $\overline{BA_1C}$ after the previous claim. $\blacksquare$

To finish, since $\overline{DMA}$ was tangent to the circumcircle of $\triangle AB_1C_1$, we have $MD^2 = MA^2 = MC_1 \cdot MB_1$, implying the required tangency.



Remark: The triangles satisfying the problem hypothesis are exactly the ones satisfying $r_A = 2R$, where $R$ and $r_A$ denote the circumradius and $A$-exradius.



Remark: If $P$ is the foot of the $A$-altitude then this should also imply $AB_1PC_1$ is harmonic.

Second solution by inversion and mixtilinears (Anant Mudgal) As in the end of the preceding solution, we have $\angle AB_1A_1=\angle AC_1A_1=90^{\circ} \quad\text{and}\quad I \in \overline{B_1 C_1}$. Let $M$ be the midpoint of minor arc $BC$ and $N$ be the midpoint of arc $\widehat{BAC}$. Let $L$ be the intouch point on $\overline{BC}$. Let $O$ be the circumcenter of $\triangle ABC$. Let $K=\overline{AI} \cap \overline{BC}$.



[asy]  /* Geogebra to Asymptote conversion, documentation at artofproblemsolving.com/Wiki, go to User:Azjps/geogebra */ import graph; size(12cm);  real labelscalefactor = 0.5; /* changes label-to-point distance */ pen dps = linewidth(0.7) + fontsize(10); defaultpen(dps); /* default pen style */  pen dotstyle = black; /* point style */  real xmin = -4., xmax = 6., ymin = -3., ymax = 4.;  /* image dimensions */

/* draw figures */ draw(circle((2.7,0.6), 2.7658633371878665), linewidth(0.4));  draw((1.892579421398536,1.2)--(3.5074205786014643,0.), linewidth(0.4) + dotted);  draw((1.445226775603362,3.064861893765504)--(-3.109814760356735,0.), linewidth(0.4));  draw((-3.109814760356735,0.)--(2.7,-2.1658633371878664), linewidth(0.4));  draw((0.8894305730917524,2.690894150918487)--(3.9547732243966385,-1.864861893765504), linewidth(0.4) + linetype("2 2"));  draw((1.445226775603362,3.064861893765504)--(0.,0.), linewidth(0.4));  draw((1.445226775603362,3.064861893765504)--(5.4,0.), linewidth(0.4));  draw((5.4,0.)--(-3.109814760356735,0.), linewidth(0.4));  draw((-3.5571674061519136,1.864861893765503)--(2.7,3.3658633371878666), linewidth(0.4));  draw((1.445226775603362,3.064861893765504)--(2.7,-2.1658633371878664), linewidth(0.4));  draw((-3.5571674061519136,1.864861893765503)--(-3.109814760356735,0.), linewidth(0.4));  draw((-3.109814760356735,0.)--(1.892579421398536,1.2), linewidth(0.4));  draw((1.892579421398536,1.2)--(1.892579421398536,0.), linewidth(0.4) + dotted);  draw((2.7,3.3658633371878666)--(2.7,-2.1658633371878664), linewidth(0.4));  draw((0.8894305730917524,2.690894150918487)--(0.8894305730917507,-1.4908941509184854), linewidth(0.4) + linetype("2 2"));  draw((2.7,3.3658633371878666)--(0.8894305730917507,-1.4908941509184854), linewidth(0.4) + linetype("2 2"));  draw((-3.5571674061519136,1.864861893765503)--(4.217574445111032,0.9163536869921585), linewidth(0.4));  draw((1.445226775603362,3.064861893765504)--(0.8894305730917507,-1.4908941509184854), linewidth(0.4) + linetype("2 2"));  draw((3.5074205786014643,0.)--(4.217574445111032,0.9163536869921585), linewidth(0.4) + dotted);  draw((0.6380284948350587,1.353053551156462)--(3.5074205786014643,0.), linewidth(0.4) + dotted);  draw((0.8894305730917524,2.690894150918487)--(2.7,-2.1658633371878664), linewidth(0.4) + linetype("2 2"));   /* dots and labels */ dot((0.,0.),linewidth(4.pt) + dotstyle);  label("$B$", (-0.32,-0.42), NE * labelscalefactor);  dot((2.7,0.6),linewidth(4.pt) + dotstyle);  label("$O$", (2.78,0.76), NE * labelscalefactor);  dot((2.7,-2.1658633371878664),linewidth(4.pt) + dotstyle);  label("$M$", (2.78,-2.), NE * labelscalefactor);  dot((1.892579421398536,1.2),linewidth(4.pt) + dotstyle);  label("$I$", (1.5,1.32), NE * labelscalefactor);  dot((1.445226775603362,3.064861893765504),linewidth(4.pt) + dotstyle);  label("$A$", (1.52,3.22), NE * labelscalefactor);  dot((-3.109814760356735,0.),linewidth(4.pt) + dotstyle);  label("$D$", (-3.22,-0.46), NE * labelscalefactor);  dot((1.892579421398536,0.),linewidth(4.pt) + dotstyle);  label("$L$", (1.72,-0.42), NE * labelscalefactor);  dot((4.217574445111032,0.9163536869921585),linewidth(4.pt) + dotstyle);  label("$B_1$", (4.3,1.08), NE * labelscalefactor);  dot((0.6380284948350587,1.353053551156462),linewidth(4.pt) + dotstyle);  label("$C_1$", (0.2,1.02), NE * labelscalefactor);  dot((-3.5571674061519136,1.864861893765503),linewidth(4.pt) + dotstyle);  label("$G$", (-3.48,2.02), NE * labelscalefactor);  dot((3.5074205786014643,0.),linewidth(4.pt) + dotstyle);  label("$V$", (3.38,-0.4), NE * labelscalefactor);  dot((2.7,3.3658633371878666),linewidth(4.pt) + dotstyle);  label("$N$", (2.78,3.52), NE * labelscalefactor);  dot((0.8894305730917524,2.690894150918487),linewidth(4.pt) + dotstyle);  label("$Z$", (0.36,2.5), NE * labelscalefactor);  dot((0.8894305730917507,-1.4908941509184854),linewidth(4.pt) + dotstyle);  label("$T$", (0.46,-1.78), NE * labelscalefactor);  dot((3.9547732243966385,-1.864861893765504),linewidth(4.pt) + dotstyle);  label("$A'$", (4.04,-1.7), NE * labelscalefactor);  dot((-0.8322939923766871,1.5324309468827513),linewidth(4.pt) + dotstyle);  label("$X$", (-0.98,1.74), NE * labelscalefactor);  dot((5.4,0.),linewidth(4.pt) + dotstyle);  label("$C$", (5.38,-0.36), NE * labelscalefactor);  dot((2.1804415825316923,0.),linewidth(4.pt) + dotstyle);  label("$K$", (2.2,-0.4), NE * labelscalefactor);  clip((xmin,ymin)--(xmin,ymax)--(xmax,ymax)--(xmax,ymin)--cycle);   /* end of picture */ [/asy]



Claim: We have $\angle (\overline{AI}, \overline{B_1C_1})=\angle IAD$.



Proof. Let $Z$ lie on $(ABC)$ with $\angle AZI=90^{\circ}$. By radical axis theorem on $(AIZ), (BIC),$ and $(ABC)$, we conclude that $D$ lies on $\overline{AZ}$. Let $\overline{NI}$ meet $(ABC)$ again at $T \ne N$.

Inversion in $(BIC)$ maps $\overline{AI}$ to $\overline{KI}$ and $(ABC)$ to $\overline{BC}$. Thus, $Z$ maps to $L$, so $Z, L, M$ are collinear. Since $BL=CV$ and $OI=OV$, we see that $MLIN$ is a trapezoid with $\overline{IL} \parallel \overline{MN}$. Thus, $\overline{ZT} \parallel \overline{MN}$.

It is known that $\overline{AT}$ and $\overline{AA_1}$ are isogonal in angle $BAC$. Since $\overline{AV}$ is a circumdiameter in $(AB_1C_1)$, so $\overline{AT} \perp \overline{B_1C_1}$. So $\measuredangle ZAI=\measuredangle NMT=90^{\circ}-\measuredangle TAI=\measuredangle (\overline{AI}, \overline{B_1C_1})$. $\blacksquare$

Let $X$ be the midpoint of $\overline{AD}$ and $G$ be the reflection of $I$ in $X$. Since $AIDG$ is a rectangle, we have $\measuredangle AIG=\measuredangle ZAI=\measuredangle (\overline{AI}, \overline{B_1C_1})$, by the previous claim. So $\overline{IG}$ coincides with $\overline{B_1C_1}$. Now $\overline{AI}$ bisects $\angle B_1AC_1$ and $\angle IAG=90^{\circ}$, so $(\overline{IG}; \overline{B_1C_1})=-1$.

Since $\angle IDG=90^{\circ}$, we see that $\overline{DI}$ and $\overline{DG}$ are bisectors of angle $B_1DC_1$. Now $\angle XDI=\angle XID \implies \angle XDC_1=\angle XID-\angle IDB_1=\angle DB_1C_1$, so $\overline{XD}$ is tangent to $(DB_1C_1)$.
Z K Y
The post below has been deleted. Click to close.
This post has been deleted. Click here to see post.
jbaca
225 posts
#13 • 3 Y
Y by AlastorMoody, Adventure10, Rounak_iitr
Solution. As usual, let $I_X$ be the $X$-excenter of $\bigtriangleup ABC$. It's known that $I_AA_1,I_BB_1$ and $I_CC_1$ concur at the circumcenter $K$ of $\bigtriangleup I_AI_BI_C$. Since $\angle AC_1I_C=90^\circ=\angle I_BB_1A$, $K$ lies on $(AB_1A_1C_1)$. Moreover, $A_1$ and $K$ lie on the same side with respect to $B_1C_1$, therefore that $K=A_1$.
Let's start with the following result, which doesn't depend of the cyclicity of $AC_1A_1B_1$.

Lemma 1. Let $ABC$ a non-isosceles triangle and $A_1$ as defined in the original problem. Let $M$ be the midpoint of $\widehat{BAC}$ and $H=\overline{MI}\cap \overline {BC}$. Then $AMA_1H$ is cyclic.

Proof. $T=\overline{MI}\cap (ABC)$ is the $A$-mixtilinear intouch point. Lines $AT$ and $AA_1$ are known to be the reflections of each other across the internal bisector of $\angle BAC$. Hence,
$$\angle AMH=\angle AMT=\angle ACB+\angle BAT=\angle ACA_1+\angle A_1AC=\angle AA_1H$$as required. $\blacksquare$

Now, let's prove the following.
Claim 1. $B_1, I $ and $C_1$ lie on a same line.

Proof. Observe that $M$ is the second point of intersection of $(ABC)$ and $(AB_1A_1C_1)$, so it carries $\overline{BC}$ to $\overline{C_1B_1}$ and thus $MB_1=MC_1$. Define $H$ to be the $A$-foot of altitude on $BC$, which clearly lies on $(AB_1A_1C_1)$. By lemma 1, $H,\ I$ and $M$ are collinear. Now, $-1=(B,A;I_C,I_B)\overset{A}{=}(H,A;C_1,B_1)$ implying that $AC_1HB_1$ is a harmonic quadrilateral, so the internal bisectors of $\angle C_1HB_1$ and $\angle C_1AB_1$ meet at a point $I'$ on $C_1B_1$. Because $M$ is the midpoint of $\widehat{C_1AB_1}$ we conclude that $I'$ is the intersection point of $AI$ and $MH$, which forces $I=I'$ to happen. Hereby, $C_1,\ I$ and $B_1$ are collinear. $\blacksquare$

Claim 2. $DI$ bisects $\angle C_1DB_1$.

Proof. Let $F=\overline{I_CI_B}\cap\overline{C_1B_1}$. We have $(F,I;C_1,B_1)=-1$ so $\angle FHI=90^\circ$ and $AIHF$ is cyclic. Further, $\angle AID=90^\circ=\angle AHD$, so $AIHD$ is inscribed. We infer that $AIHDF$ is a cyclic pentagon, implying that $D$ lies on the $A$- Apollonius circle of $\bigtriangleup AB_1C_1$, from which the result follows. $\blacksquare$

Finally, note that $\angle IDF=\angle IHF=90^\circ$, then,
$$\angle ADC_1=90^\circ-\angle ADF-\angle C_1DI=90^\circ-\angle AIF-\angle B_1DI=\angle C_1ID-\angle B_1DI=\angle C_1B_1D$$thus, $DA$ is tangent to $(B_1DC_1)$. $\blacksquare$
Attachments:
Z K Y
The post below has been deleted. Click to close.
This post has been deleted. Click here to see post.
dangit
32 posts
#14 • 2 Y
Y by Adventure10, Mango247
Here s another solution. It seems a little longer, and not so straight-forward maybe, but I think it reveals more interesting properties of the configuration.
Firslty, take $Q$ the second intersection of $(AB_1C_1)$ and $BC$. Notice that the perpendiculars raised from $A_1,B_1,C_1$ onto $BC,CA,AB$ meet at some point. Recall that if $P$ and $P'$ are isogonal conjugates WRT $ABC$ the projections of $P,P'$ to the sides of the triangle are concyclic. Conversely, taking the limit case, The perpendiculars raised from $A$ onto $AB$, $A$ onto $AC$ and $Q$ onto $BC$ meet at some point, that must be $A$! Hence, $Q$ is the foot of the altitude from $A$ to $BC$, and $(AB_1C_1)$ is the circle of diameter $AA_1$. Also note that the midpoin $M$ of $\overarc{BAC}$ in $(ABC)$ generally lays on circle $(AB_1C_1)$. Denote now $I_a,I_b,I_c$ the three center of the excircles, noticing that $A_1, I_a, B_1$, respectively $A_1, C_1, I_c$ are collinear. Now, from quadrilateral $B_1A_1C_1M$, as $MB_1=MC_1$ it follows that $(A_1M$ bisect $\angle{B_1A_1C_1}$ but $A_1M\perp{I_bI_c}$, so $A_iI_bI_c$ is isosceles, and also $m(\angle{I_bA_1I_c})=2m(\angle{I_bI_aI_c})$ so $A1$ is the Bevan point of $ABC$. It follows that $O$ is the midpoint of $A_1I$. Taking $M'$ the midpoint of $\overarc{BC}$ in circle $(ABC)$ it follows that $MA_1M'I$ is a parallelogram. To prove $M,I,Q$ collinear, it suffices to prove $\frac{AQ}{AI}=\frac{MM'}{MI}$, which reduces to $2\sin{\frac{A}{2}}\cos{\frac{B}{2}}\cos{\frac{C}{2}}=1$ (which i think is a nice way to characterise such triangles). To prove this, using Pappus for $B,A_1,C$ and $I_c, A,I_b$ it follows that $B_1,I,C_1$ are collinear. Using now a theorem(whose name i do not know in English) it follows that $BA_2\cdot{\frac{CB_1}{AB_1}}+CA_2\cdot{\frac{BC_1}{C_1A}}=BC\cdot{\frac{A_2I}{IA}}$, where $A_2\in{ BC\cap{AI}}$, which,after some computations, finally yields the desired $2\sin{\frac{A}{2}}\cos{\frac{B}{2}}\cos{\frac{C}{2}}=1$, and $M,I,Q$ collinear. As $M$ is the midpoint of $I_bI_c$ and $ID||I_bI_c$ it follows that $(II_b,II_c;IM,ID)=-1 \Rightarrow (II_b\cap{BC},II_c\cap{BC};IM\cap{BC},ID\cap{BC})=-1 \Rightarrow (B,C;Q,D)=-1$
Now, take $B_2\in{DC_1\cap{AC}}, C_2\in{DB_1\cap{AB}}, R\in{C_2B_2\cap{C_1B_1}}, S\in{AB\cap{DR}}, T\in{AC\cap{DR}}, U\in{AQ\cap{DB_1}}, V\in{AQ\cap{DC_1}}$. $|angle{AQD}$ is right and $(D,U;C_2,B_1)+(D,V;C_1,B_2)=-1$ so $(QA$ bisects both $\angle{C_2QB_1}$ and $C_1QB_2$, which implies that $AB_2QC_2$ is cyclic. This means $Q$ is the Miguel point of $C_2C_1B_1B_2$, but them $QRC_1C_2, QRB_2B_2$ are cyclic as well. We have that $(D,R;S,T)=-1$ so $D,AR\cap{BC};B,C)=-1$ hence $R\in{AQ}$. Using now the power of $A$ WRT $CRC_1, QRB_1$ it follows that $C_1B_2B_1C_2$ is cyclic. Doing some angle chasing, the desired tangency is equivalent to $B_2C_2||AD$, whyich however i was not able to prove staight-forwar :((, so, even more $B_2C_2||AD$ is equivalent to $AD$ tangent to $(AC_1B_1)$. let $N\in{B_1C_1\cap{AD}}$, then $(N,R;C_1,B_1)=-1$ and it is enough to prove that $AR$ is symmedian in $AC_1B_1$, which is equivalent to $(\frac{AC_1}{AB_1})^2=\frac{C_1R}{B_1R}$. But $\frac{C_1R}{RB_1}=\frac{AC_1\sin{QAC_1}}{AB_1\sin{QAB_1}}$, so tha tangency actually resumes to $\frac{\cot{\frac{B}{2}}}{\cot{\frac{C}{2}}}=\frac{\cos{B}}{\cos{C}}$, which comes down, once again, to $2\sin{\frac{A}{2}}\cos{\frac{B}{2}}\cos{\frac{C}{2}}=1$, which, after laborious work, ends the proof.
Z K Y
The post below has been deleted. Click to close.
This post has been deleted. Click here to see post.
meysam1371
35 posts
#15 • 2 Y
Y by Adventure10, Mango247
Following are equivalent:
1. $r_{a}=2R$;
2. $\frac{a}{p-a}=\frac{b}{p-b}+\frac{c}{p-c}$;
3. $\frac{1}{p}+\frac{1}{p-a}=\frac{1}{p-b}+\frac{1}{p-c}$;
4. $O$ is midpoint of $IA_{1}$;
and other facts could be driven from figure of problem which had been noticed by other matlinkers!
Z K Y
The post below has been deleted. Click to close.
This post has been deleted. Click here to see post.
TheUltimate123
1740 posts
#17 • 4 Y
Y by AlastorMoody, nixon0630, Adventure10, Rounak_iitr
Let $X$ be the antipode of $A$ on $(AB_1C_1)$. Let the incircle of $\triangle ABC$ touch $\overline{CA}$ and $\overline{AB}$ at $E$ and $F$, respectively, and let $J$ be the Miquel point of $BCEF$. Furthermore, let $M$ be the midpoint of $\overline{AD}$.
[asy]
size(10cm);
defaultpen(fontsize(10pt));

pen pri=royalblue+linewidth(0.5);
pen sec=rgb(41, 207, 255)+linewidth(0.5);
pen tri=springgreen+linewidth(0.5);
pen qua=chartreuse+linewidth(0.5);
pen fil=invisible;
pen sfil=invisible;
pen tfil=invisible;
pen qfil=invisible;

pair B, C, L, I, A, A1, B1, C1, EE, F, J, D, M, K, IB, IC;
B=dir(190); C=dir(350); L=dir(270);
I=intersectionpoint(arc(L, length(B-L), 90, 180, CCW), (-B) -- (-C));
A=intersectionpoint(I -- (I+100*(I-L)), circle((0, 0), 1));
A1=B+C-foot(I, B, C);
B1=foot(A1, A, C);
C1=foot(A1, A, B);
EE=foot(I, A, C);
F=foot(I, A, B);
J=intersectionpoints(circumcircle(A, B, C), circumcircle(A, EE, F))[1];
D=extension(A, J, B, C);
M=(A+D)/2;
K=dir(90);
IB=extension(B, I, A, K);
IC=extension(C, I, A, K);

draw(A -- B -- C -- A, pri);
filldraw(circumcircle(A, B, C), fil, pri);
draw(IB -- A1 -- IC -- IB, sec);
filldraw(circumcircle(A, B1, C1), tfil, tri);
draw(B -- D -- A, pri);
filldraw(circumcircle(A, EE, F), sfil, sec);
filldraw(circumcircle(B, I, C), qfil, qua);
draw(B -- IB, tri+dashed); draw(C -- IC, tri+dashed);
draw(M -- B1, pri);
draw(D -- I -- J, sec); draw(A -- L, pri);

clip((L + (100, -1/8)) -- (L+(-100, -1/8)) -- (-100, 100) -- (100, 100) -- cycle);

dot("$A$", A, NW);
dot("$B$", B, SW);
dot("$C$", C, SE);
dot("$L$", L, S);
dot("$I$", I, NE);
dot("$A_1$", A1, SE);
dot("$B_1$", B1, NE);
dot("$C_1$", C1, SW);
dot("$E$", EE, NE);
dot("$F$", F, NW);
dot("$J$", J, dir(150));
dot("$D$", D, SW);
dot("$M$", M, dir(195));
dot("$K$", K, N);
dot("$I_B$", IB, NE);
dot("$I_C$", IC, W);
[/asy]
We claim that $\overline{AA_1}$ is a diamter of $(AB_1A_1C_1)$. Note that $X$ is the Bevan point of $\triangle ABC$, so $\overline{XA_1}\perp\overline{BC}$. Furthermore, if $X\ne A_1$, then $\overline{XA_1}\perp\overline{AA_1}$, which would require that $A\in\overline{BC}$, which is absurd.

By Pappus' Theorem on $\overline{BA_1C}$ and $\overline{I_CAI_B}$, $I$ lies on $\overline{B_1C_1}$, and by the Radical Axis Theorem on $(AI)$, $(ABC)$, and $(BIC)$, $J$ lies on $\overline{AD}$. Since $\triangle JBF\sim\triangle JCE$, $$\frac{AC_1}{AB_1}=\frac{JF}{JE}=\frac{JB}{JC},$$and also $\measuredangle C_1AB_1=\measuredangle BAC=\measuredangle BJC$, so $\triangle AC_1B_1\sim\triangle JBC$. This implies that $$\measuredangle DAB=\measuredangle JAF=\measuredangle JEF=\measuredangle JCB=\measuredangle AB_1C_1,$$so $\overline{AD}$ is tangent to $(AB_1C_1)$. Moreover, $$\measuredangle MIA=\measuredangle IAM=\measuredangle IAC_1+\measuredangle C_1AM=\measuredangle B_1AI+\measuredangle C_1B_1A=\measuredangle B_1IA,$$whence $M$ lies on $\overline{B_1IC_1}$. Hence, $MD^2=MA^2=MB_1\cdot MC_1$, and the desired result follows. $\square$
This post has been edited 2 times. Last edited by TheUltimate123, May 25, 2019, 6:38 AM
Z K Y
The post below has been deleted. Click to close.
This post has been deleted. Click here to see post.
yayups
1614 posts
#18 • 2 Y
Y by Adventure10, Mango247
I think I'm getting worse at synthetic geometry as I wasn't able to solve this one. However, I can still bash and that worked out :D

https://lh3.googleusercontent.com/-KkqkgIb7Xhg/XO4AegmYGhI/AAAAAAAAFN8/C30kYHUBXcYlA8uwfLuAi0LFTTcv-h7RwCK8BGAs/s0/2019-05-28.jpg

It turns out that this problem can be bashed mostly with barycentric coordinates, with only some basic synthetic insights.

It's easy to see that $(AB_1C_1)=(AV)$ where $V$ is the Bevan point, so either $V=A_1$, or $\angle AA_1V=90^\circ$. But $I_A,A_1,V$ collinear, so we have $\angle AA_1I_A=90^\circ$. But $I_AA_1\perp BC$, so we have $BC\parallel AA_1$, which is nonsense. Thus, $V=A_1$, so the Bevan point is on $BC$.

Thus, we have $I_BB_1$, $I_CC_1$, and $BC$ concur. In barycentric coordinates, let $V=(0:y:z)=I_CC_1\cap BC$. We see then that
\[\begin{vmatrix}0 & y & z \\ a & b & -c \\ s-a & s-b & 0\end{vmatrix}=0,\]which gives $V=(0:s(a-b):c(s-a))$. Similarly, we get $V=(0:b(s-a):s(a-c))$, so for these to be equal, we must have
\[s^2(a-b)(a-c)=bc(s-a)^2.\]We see that
\begin{align*}
s^2(a-b)(a-c)=bc(s-a)^2 &\iff (a+b+c)^2(a-b)(a-c) = bc(s-a)^2 \\
&\iff bc\left[(a+b+c)^2-(b+c-a)^2\right]+a^2(a+b+c)^2 = a(b+c)(a+b+c)^2 \\
&\iff bc[2(b+c)][2a]=a(a+b+c)^2(b+c-a) \\
&\iff 4bc(b+c)=(a+b+c)^2(b+c-a) \\
&\iff \boxed{bc(b+c)=2s^2(s-a)}.
\end{align*}We now compute $D=(0,y,z)$. We can use the perpendicularity criterion for $AI\perp ID$, but that's ugly. A better way is to define $K\in BC$ such that $AK\parallel ID$. We see that $AK$ is the external angle bisector which is parameterized by $(t:b:-c)$ (an easy way to see this is that $I_C=(a:b:-c)$ is on it), so $K=(0:b:-c)=\left(0,\frac{b}{b-c},\frac{c}{b-c}\right)$. We have $AK\parallel ID$, so
\[[AKI]=[AKD]\implies \frac{1}{(b-c)(a+b+c)}\begin{vmatrix}1 & 0 & 0\\ 0 & b & -c \\ a & b & c\end{vmatrix}=\frac{1}{b-c}\begin{vmatrix}1 & 0 & 0\\ 0 & b & -c \\ 0 & y & z\end{vmatrix}.\]This implies $bz+cy=\frac{2bc}{a+b+c}$, which combined with $y+z=1$ can be solved to give
\[\boxed{D=\left(0,\frac{b(s-c)}{b-c},-\frac{c(s-b)}{b-c}\right)=(0:b(s-c):-c(s-b))}.\]Let $E$ be the midpoint of $AD$, which by the above is given by
\[E=(s(b-c):b(s-c):-c(s-b)).\]The first big claim is that $E,B_1,C_1$ are collinear. This is equivalent to
\[\begin{vmatrix}s(b-c) & b(s-c) & -c(s-b) \\ s-a & s-b & 0 \\ s-a & 0 & s-c\end{vmatrix}=0,\]which is equivalent to
\[\Gamma:=s(b-c)(s-b)(s-c)-(s-a)b(s-c)^2+(s-a)c(s-b)^2=0.\]We have
\begin{align*}
\Gamma=0 &\iff s(b-c)(s-b)(s-c)=(s-a)[b(s-c)^2-c(s-b)^2] \\
&\iff s(b-c)(s-b)(s-c)=(s-a)(b-c)(s^2-bc) \\
&\iff s(s-b)(s-c)=(s-a)(s^2-bc) \\
&\iff s^3-s^2(b+c)+sbc=s^3-sbc-as^2+abc \\
&\iff bc(2s-a)=s^2(b+c-a) \\
&\iff bc(b+c)=2s^2(s-a),
\end{align*}which as we saw before, is directly equivalent to $(AB_1A_1C_1)$ cyclic. Thus, $E,C_1,B_1$ are collinear.

The next big claim is that $AD$ is tangent to $(AB_1C_1)$. Doing this directly is difficult, but after performing a $\sqrt{bc}$ inversion, it is quite susceptible to barycentric coordinates. Let $D'$ be on $BC$ such that $AD$ and $AD'$ are isogonal. Given that $D=(0:b(s-c):-c(s-b))$, we have that
\[D'=(0:b(s-c):-c(s-c)),\]recalling that $(0:y:z)\mapsto(0:b^2/y:c^2/z)$. Let $B_1'\in AB$ be the $\sqrt{bc}$ inverse of $B_1$, and let $C_1'\in AC$ be the $\sqrt{bc}$ inverse of $C_1$. We have that
\[AB_1\cdot AB_1'=bc,\]so $AB_1'=\frac{bc}{s-c}$. Thus,
\[BB_1'=c-\frac{bc}{s-c}=c\frac{s-b-c}{s-c}=-\frac{c(s-a)}{s-c},\]so
\[B_1'=(BB_1':AA_1':0)=(a-s:b:0),\]and similarly $C_1'=(a-s:0:c)$. Note that $(a-s)+b=c-s$ and $(a-s)+c=b-s$.

The fact that $AD$ is tangent to $(AB_1C_1)$ is equivalent to $AD'\parallel B_1'C_1'$ under the $\sqrt{bc}$ inversion, which is equivalent to
\[[AD'B_1']=[AD'C_1'].\]This is equivalent to
\[\frac{1}{c-s}\begin{vmatrix}1 & 0 & 0\\ 0 & b(s-b) & -c(s-c) \\ a-s & b & 0\end{vmatrix}=\frac{1}{b-s}\begin{vmatrix}1 & 0 & 0\\ 0 & b(s-b) & -c(s-c) \\ a-s & 0 & c\end{vmatrix},\]which is immediate upon expansion as they are both equal to $-bc$. Note that we proved $DA$ tangent to $(AB_1C_1)$ independent of the problem condition, so it's true for any $\triangle ABC$.

Thus, we have $EB_1C_1$ collinear where $E$ is the midpoint of $AD$, and we have $DA$ tangent to $(AB_1C_1)$. Thus,
\[EA^2=EB_1\cdot EC_1,\]so $ED^2=EB_1\cdot EC_1$ as $EA=ED$. This directly implies $ED$ tangent to $(DB_1C_1)$, or $AD$ tangent to $(DB_1C_1)$, as desired.
This post has been edited 1 time. Last edited by yayups, May 29, 2019, 3:48 AM
Reason: fixed align*
Z K Y
The post below has been deleted. Click to close.
This post has been deleted. Click here to see post.
v_Enhance
6877 posts
#19 • 12 Y
Y by yayups, pad, a_simple_guy, Kayak, anantmudgal09, AlastorMoody, Euler1728, amar_04, v4913, Aimingformygoal, khina, Adventure10
yayups wrote:
I think I'm getting worse at synthetic geometry as I wasn't able to solve this one.
It's TST6, don't be too hard on yourself ;) you'll get some practice in June anyways.
Z K Y
The post below has been deleted. Click to close.
This post has been deleted. Click here to see post.
tworigami
844 posts
#21 • 1 Y
Y by Adventure10
This problem generalizes a little
Quote:
Let $ABC$ be a triangle and let $A_1$ be a point on $\overline{BC}$. Let $B_1$ and $C_1$ be the projections of $A_1$ onto $\overline{AC}$ and $\overline{AB}$, respectively. Let $B_2$ be the reflection of $B_1$ over the perpendicular bisector of $\overline{AC}$ and define $C_2$ similarly. Let the radical axis of $\odot(ABC)$ and $\odot(A_2B_2C_2)$ meet $\overline{BC}$ at $D$.

Given that $\overline{AA_1}, \overline{BB_1}, \overline{CC_1}$ are concurrent, prove that the circumcircle of $\triangle B_1C_1D$ is tangent to $\overline{AD}$.
Z K Y
The post below has been deleted. Click to close.
This post has been deleted. Click here to see post.
Idio-logy
206 posts
#22 • 1 Y
Y by Adventure10
Nice problem with lots of good properties. We list some of them and prove them:
Claim 1. Let $\omega = (AA_1B_1C_1)$. We have that $AA_1$ is a diameter of $\omega$. (Hence if $AH\perp BC$ at $H$, then $H$ also lies on $\omega$.)
Claim 2. Let $K$ be the tangency point of the $A$-mixtilinear circle, then $KH$ bisects $\angle BKC$. This immediately implies that $K,H,I$ are collinear.
Claim 3. Let $\gamma=(AID)$. Then $\gamma$ and $\omega$ are orthogonal.
Claim 4. Let $M$ be the midpoint of $AD$, then $I,M,B_1,C_1$ are collinear. This implies the desired conclusion.

Proof of Claim 1. Let $I'$ be the reflection of $I$ over the circumcenter of $ABC$ (i.e. the Bevan point). Then $I'C_1\perp AB$, $I'B_1\perp AC$, $I'A_1\perp BC$. Thus $I'$ lies on $\omega$, forcing $I'=A$.
Proof of Claim 2. Observe $\angle KBC=\angle KAC=\angle BAA_1=\angle C_1B_1A_1$, so $\triangle A_1B_1C_1\sim\triangle KBC$. Then $$\frac{BH}{HC} = \frac{BA}{AC}\cdot\frac{\cos B}{\cos C} = \frac{\sin C}{\sin B} \cdot \frac{CA_1}{BA_1}=\frac{B_1A_1}{A_1C_1}=\frac{BK}{KC}.$$Proof of Claim 3. Let $J$ be the midpoint of arc $BAC$. We know that $K,H,I,J$ are collinear, so $\angle AHI=\angle AHJ$. Thus $\angle A_1AI=\frac12 \angle A -\angle A_1AC= \angle AC_1J=\angle AHI$, meaning that $AA_1$ is tangent to $\gamma$.
Proof of Claim 4. By angle chasing, $\angle MIA = \frac{\pi}{2} - \angle IAA_1=\angle AIC_1 = \pi - \angle AIB_1$.
Attachments:
This post has been edited 1 time. Last edited by Idio-logy, Dec 9, 2019, 10:05 PM
Z K Y
The post below has been deleted. Click to close.
This post has been deleted. Click here to see post.
Th3Numb3rThr33
1247 posts
#23
Y by
Label all of the following points:
  • The excenters $I_B$ and $I_C$.
  • The midpoint $O$ of $\overline{AD}$ (so the center of $(AID)$).
  • The foot $E$ of $A$ onto $\overline{BC}$.
  • The point $X = AM_A \cap B_1C_1$.
  • The $A$-mixtillinear touchpoint $T$.

In typical ``American geometry" fashion, we now prove many lemmas.

Claim 1. Quadrilateral $AM_AB_1C_1$ is cyclic.

Proof. Take the spiral similarity at $M_A$ sending $BC_1 \to CB_1$, which is actually a rotation as $BC_1 = CB_1$. So $BM_A = CM_A$ and the result follows. $\square$

Claim 2 (EGMO TST 2019/5). Circle $(AB_1A_1C_1)$ has diameter. In addition, $B_1, I, C_1$ are collinear.

Proof. Lines $I_BB_1$ and $I_CC_1$ meet at the Bevan point $V$ with $VA_1 \perp BC$. However, this implies $V=A_1$, as desired. The second part follows from Pappus on $\overline{I_CAI_B}$ and $\overline{BA_1C}$. $\square$

Claim 3. Point $E$ lies on $\overline{TIM_A}$.

Proof. Note
\[\measuredangle AM_AT = \measuredangle TAI + \measuredangle IAC + \measuredangle CBA = \measuredangle IAA_1 + 90^\circ + \measuredangle EAI = \measuredangle AM_AE,\]as desired. $\square$

Claim 4. Line $AA_1$ is tangent to $(AID)$.

Proof. Note $(AB_1A_1C_1)$ and $(AID)$ both pass through $E$. By the first concylicitiy, it suffices to show $\measuredangle AEI = \measuredangle A_1AI \measuredangle IAT$, or $\measuredangle IAE = \measuredangle ATI = \measuredangle ATM_A$, which follows from simple angle chasing (since both of these are known angles in terms of the angles of the triangle). $\square$

Claim 5. Point $O$ lies on $\overline{B_1IC_1}$.

Proof. It suffices to show lies $AM_A$ and $B_1C_1$ meet on $(AID)$, or $\angle AXI = \measuredangle AEI = \measuredangle IAT$, or $AT \perp B_1C_1$. This follows from $\overline{AA_1}, \overline{AT}$ isogonal in $\angle B_1AC_1$. $\square$

We are now done, since $OD^2 = OA^2 = OB_1 \cdot OC_1$ and so $\overline{OD}$ is tangent to $(DB_1C_1)$. $\blacksquare$

[asy]
size(12cm);
defaultpen(fontsize(10pt));

pair A = dir(105.3); pair B = dir(193.3); pair C = dir(346.7);
pair I = incenter(A,B,C);
pair A1 = B+C-foot(I,B,C);
pair B1 = A+C-foot(I,A,C);
pair C1 = A+B-foot(I,A,B);
pair D = extension(I,I+(A-I)*dir(90), B, C);
pair Ma = intersectionpoints(circumcircle(A1,B1,C1),circumcircle(A,B,C))[0];
pair Ic = extension(A,Ma,A1,C1); pair Ib = extension(A,Ma,A1,B1);
pair E = foot(A,B,C);
pair O = (A+D)/2;
pair X = 2O-I;
pair T = intersectionpoints(Ma--3I-2Ma, circumcircle(A,B,C))[0];

filldraw(A--B--C--cycle,orange+white+white,orange);

filldraw(anglemark(I,E,A),orange,orange);
filldraw(anglemark(I,A,A1),orange,orange);
filldraw(anglemark(I,X,A),orange,orange);

draw(circumcircle(A,B,C), red);
draw(circumcircle(A1,B1,C1), red);
draw(circle((A+D)/2,abs(A-D)/2), red);
draw(X--Ib, red+white);
draw(D--B, orange);
draw(X--B1, red+white+dashed);
draw(D--I--A,red);
draw(Ma--T, red+white);
draw(Ic--A1--Ib, red+white+dashed);
draw(A--A1, red+white);
draw(A--T, red+white);
draw(A--E, red+white);


dot("$A$", A, dir(135));
dot("$B$", B, dir(225));
dot("$C$", C, dir(-45));
dot("$I$", I, dir(45));
dot("$A_1$", A1, dir(270));
dot("$B_1$", B1, dir(0));
dot("$C_1$", C1, dir(180));
dot("$D$", D, dir(180));
dot("$M_A$", Ma, dir(90));
dot("$I_B$", Ib, dir(90));
dot("$I_C$", Ic, dir(90));
dot("$E$", E, dir(-90));
dot("$O$", O, dir(90));
dot("$X$", X, dir(135));
dot("$T$", T, dir(-90));
[/asy]
Z K Y
The post below has been deleted. Click to close.
This post has been deleted. Click here to see post.
mathlogician
1051 posts
#24 • 1 Y
Y by Mango247
Claim: [USA EGMO TST 2019/5]

Draw the excenters $I_A,I_B,I_C$ and let $V$ be the circumcircle of $\triangle I_AI_BI_C$. Note that $\overline{A_1I_A},\overline{B_1I_B},\overline{C_1I_C}$ concur at $V$. Now $$\angle C_1A_1B = 90^{\circ} - \angle C_1BA_1, \angle B_1A_1C = 90^{\circ} - \angle A_1CB_1, \angle C_1A_1B_1 = 180^{\circ} - \angle C_1AB_1,$$so $V$ lies on $\overline{BC}$, and thus $V = A_1.$ Now $\overline{A_1B_1I_B}$ and $\overline{A_1C_1I_C}$ are collinear, so by Pappus $\overline{C_1IB_1}$ is collinear.
Let $\overline{AP}$ be an altitude in $\triangle ABC,$ and let the tangents of $(A_1B_1C_1)$ at $A$ and $P$ intersect at $X.$ Note that $X$ lies on $\overline{B_1C_1}$ as $-1 = (\overline{I_BI_C} \cap \overline{BC},A; I_B,I_C) \stackrel{A_1}{=} (P,A;B_1,C_1),$ so $\overline{AP}$ is a symmedian in $\triangle AC_1B_1.$ This implies that $(AIP)$ is the Apollonian circle of $\overline{B_1C_1}$ passing through $A$. Thus $X$ is the midpoint of $\overline{AD},$ and we conclude that $XD^2 = XA^2 = XB_1 \cdot XC_1.$

Remarks: This was a nice problem (and there seems to be a trend with Ankan geometry diagrams that are hard to draw; USAMO 2019/2, TSTST 2019/9, and TST 2019/6). I wasn't able to find the last step of using Apollonian circles, unfortunately.
This post has been edited 2 times. Last edited by mathlogician, Sep 19, 2020, 7:13 PM
Reason: wrong tstst number
Z K Y
The post below has been deleted. Click to close.
This post has been deleted. Click here to see post.
Orestis_Lignos
558 posts
#25
Y by
USA 2019 TST P6 wrote:
Let $ABC$ be a triangle with incenter $I$, and let $D$ be a point on line $BC$ satisfying $\angle AID=90^{\circ}$. Let the excircle of triangle $ABC$ opposite the vertex $A$ be tangent to $\overline{BC}$ at $A_1$. Define points $B_1$ on $\overline{CA}$ and $C_1$ on $\overline{AB}$ analogously, using the excircles opposite $B$ and $C$, respectively.

Prove that if quadrilateral $AB_1A_1C_1$ is cyclic, then $\overline{AD}$ is tangent to the circumcircle of $\triangle DB_1C_1$.

Ankan Bhattacharya

Probably the longest solution in the thread :D
Anyway, I think this solution is quite different from the others.

Let $I_A, I_B$ and $I_C$ be the three excenters and let $M$ be the midpoint of $I_BI_C$. Since the circumcircle of $ABC$ is the Euler circle of $I_AI_BI_C$, point $M$ belongs to this circle and actually it is the midpoint of the arc $BAC$.
Let $O \equiv I_CC_1 \cap I_BB_1$.

We prove now several Claims:

Claim 1: $O$ is the circumcenter of triangle $I_AI_BI_C$.
Proof: Well known fact. By Nagel's theorem, the circumcenter of $I_AI_BI_C$, belongs to the perpendiculars from $I_C$ to $AB$ and from $I_B$
to $AC$, hence the result $\blacksquare$.

Claim 2: $AC_1OB_1$ is cyclic.
Proof: Obvious since $\angle AC_1O=\angle AB_1O=90^\circ$.

Claim 3: $A_1 \equiv O$
Proof: We know that $A_1$ and $O$ both belong to the circumcircle of $AB_1C_1$. In addition, by Nagel's $I_AO \perp BC$ hence $I_A,A_1,O$ are collinear. Hence $O$ and $A_1$ must necessarily coincide. $\blacksquare$.

Claim 4: $I \in B_1C_1$.
Proof: Apply Pappus at $\overline{I_CAI_B}$ and $\overline{BA_1C}$ $\blacksquare$.

Claim 5: Let $X$ be the midpoint of $B_1C_1$. Then $AX$ and $MA_1$ intersect on the antipode of $A$ on the circumcircle of $ABC$.
Proof: Define $X \equiv AY \cap B_1C_1$ where $Y \equiv MA_1 \cap (A,B,C)$. We aim to prove two things:
(i) $X$ is the midpoint of $B_1C_1$
(ii) $Y$ is the antipode of $A$.

For (i), note that $$\frac{BA_1}{CA_1}=\dfrac{BM}{MC} \cdot \dfrac{\sin \angle BMY}{\sin \angle CMY}=\dfrac{\sin \angle BAY}{\sin \angle CAY}=\dfrac{AB_1}{AC_1} \cdot \dfrac{XC_1}{XB_1},$$and since $BA_1=AB_1$ and $CA_1=AC_1$, we obtain that $XC_1=XB_1$.

For (ii), note that $$\angle BAY =\angle BMY=90^\circ-\angle I_CMB=90^\circ-\angle C,$$hence $\angle BAY+\angle AYB=90^\circ$, implying that $AY$ is the diameter, as desired $\blacksquare$.

Now, let's attack the problem.

Let $Z \equiv MI \cap BC$. We present the following angle-chase: $$\angle AA_1B-\angle AXC_1=\angle C+\angle CAA_1-\angle AB_1C_1-\angle XAB_1=\angle C+\angle CAA_1-\angle AB_1C_1-(90^\circ-\angle B)=90^\circ-\angle A+\angle CAA_1-\angle AB_1C_1=90^\circ-\angle C_1AA_1-\angle AB_1C_1=90^\circ-\angle C_1B_1A_1-\angle AB_1C_1=90^\circ-\angle AB_1A_1=0,$$therefore $\angle AA_1B=\angle AXC_1$.

Note now that since $AIXM$ is cyclic to a circle with diameter $MI$, we have $$\angle AMZ=\angle AMI=\angle AXC_1=\angle AA_1B,$$therefore $AMA_1Z$ is cyclic. This combined with the fact that $\angle AMA_1=90^\circ$ readily implies that $AZ \perp BC$.

Now, let's put point $D$ into the game.

Since $\angle AID=\angle AZD=90^\circ$, $AIZD$ is cyclic. Now, it's time for a second angle-chase: $$\angle DAB=\angle DAI-\angle BAI=\angle IZC-\angle BAI=\angle IZC-\angle IAB_1=\angle MAA_1-\angle IAB_1=\angle MC_1A_1-\angle A/2=\angle I_CMC_1=\angle AB_1C_1,$$therefore $\angle DAB=\angle AB_1C_1$, which implies that $DA$ is tangent to the circumcircle of $AB_1C_1$.

It's now time to end. Let $W \equiv B_1C_1 \cap AD$. Then, $$\angle AIW=\angle AB_1C_1+\angle A/2=\angle DAB+\angle A/2=\angle WAI,$$so $WA=WI$.
Therefore, $WA=WD=WI$, thus $WD^2=WA^2=WC_1WB_1$, giving us that $WD$ is tangent to the circumcircle of $B_1C_1D$, as desired.
Attachments:
Z K Y
The post below has been deleted. Click to close.
This post has been deleted. Click here to see post.
KST2003
173 posts
#27
Y by
Here's my unnecessarily long solution. I totally missed the Appollonian circle :( The first two claims are from USA EGMO TST 2019 P5.
Let $I_A,I_B$ and $I_C$ be the corresponding excenters. Let $M$ be the midpoint of $I_{C}I_{B}$, $N$ be the midpoint of $AD$, $P$ be the foot of perpendicular from $A$ to $BC$ and $Q$ be the intersection of $BC$ and $I_{B}I_{C}$. Also denote $(AB_{1}A_{1}C_{1})$ by $\omega$ for brevity.
Claim 1: The center of $(I_{A}I_{B}I_{C})$ coincides with $A_1$.
Proof. Assume that this is not the case, and let $O$ denote the circumcenter. Then since $\angle OC_1A=\angle OB_1A=90^\circ$, it follows that $O$ lies on $\omega$. But this means that
\[\measuredangle B_1OC_1=\measuredangle B_1A_1C_1=\measuredangle B_1OC_1-(\measuredangle OBC+\measuredangle BCO)=\measuredangle B_1OC_1+\measuredangle BOC \Rightarrow \measuredangle BOC=0.\]Therefore, $O$ lies on $BC$. But as both $I_{A}O$ and $I_{A}A_{1}$ are perpendicular to $BC$, it follows that $O\equiv A_1$.
Claim 2: $I$ lies on $C_1B_1$.
Proof. This follows by Pappus on $\overline{I_{C}AI_{B}}$ and $\overline{CBA_1}$.
Claim 3: $P$, $I$, $M$ are collinear.
Proof. Obviously, $P$ lies on $\omega$. Therefore, projecting from line $I_{B}I_{C}$ gives us
\[ (A,P;B_{1},C_{1}) \stackrel{A_1}{=} (A,Q,I_{B},I_{C})=-1\]Now notice that $M$ is the midpoint of arc $BAC$. Since $BC_1 = B_1C$, by spiral similarity, it follows that $M$ is also the midpoint of arc $B_{1}AC_{1}$ in $\omega$. Let $R$ be the midpoint of arc $A_{1}C_{1}$, and let $MI$ meet $\omega$ again at $P'$. Then
\[(A,P';B_{1},C_{1})\stackrel{I}{=}(R,M;B_{1},C_{1})=-1\]which implies that $P=P'$ as desired.
Claim 4: $AD$ is tangent to $\omega$.
Proof. Due to right angles, $A_1MAP$ and $IADP$ are concyclic. Therefore by spiral similarity, $\triangle AMI\sim\triangle A_1AD$. In particular, $AD\perp AA_1$, so $AD$ is tangent to $\omega$.
Finally, notice that $B_1C_1$, and $AD$ are concurrent on the perpendicular bisector of $AI$ as $AI$ is the internal angle bisector of $\angle B_1AC_1$. But this point is $N$, so by power of a point,
\[NB_1\cdot NC_1=NA^2=ND^2\]and it follows that $AD$ is tangent to $(DB_{1}C_{1})$.
This post has been edited 1 time. Last edited by KST2003, Jan 14, 2021, 9:46 AM
Z K Y
The post below has been deleted. Click to close.
This post has been deleted. Click here to see post.
MP8148
888 posts
#28 • 1 Y
Y by amar_04
Similar solution for storage; had to approximate the asy diagram oops.

I also discovered the following property:
Quote:
$\overline{HI}$ bisects $\overline{BC}$, where $H$ is the orthocenter of $\triangle ABC$.

[asy]
size(10cm);
defaultpen(fontsize(10pt)+linewidth(0.4));
dotfactor *= 1.2;

pair A = dir(116.8), B = dir(192.4), C = dir(347.6), I = incenter(A,B,C), Ia = 2*circumcenter(B,I,C)-I, Ib = 2*circumcenter(C,I,A)-I, Ic = 2*circumcenter(A,I,B)-I, A1 = foot(Ia,B,C), B1 = foot(Ib,C,A), C1 = foot(Ic,A,B), L = dir(90), F = foot(A,B,C), D = extension(B,C,I,I+dir(A-L)), M = (A+D)/2, T = F+dir(L--F)*abs(B-F)*abs(C-F)/abs(L-F), S = A1+dir(A--A1)*abs(B-A1)*abs(C-A1)/abs(A-A1), I1 = 2M-I;

draw(A--B--C--A, linewidth(0.8)+heavyred);
draw(unitcircle, heavyred);
draw(A--I--D--B^^F--A--D, heavyred);
draw(circumcircle(A1,B1,C1), purple);
draw(Ia--Ib--Ic--Ia, lightgreen);
draw(B--Ib--A1--Ic--C, lightgreen);
draw(circumcircle(A,D,F), heavyblue);
draw(B1--M, orange+linewidth(0.8));
draw(M--extension(A,L,B1,C1)--A, orange+dashed);
draw(L--T--S--A, magenta);
draw(arc(circumcenter(D,B1,C1),circumradius(D,B1,C1),50,135), cyan);

dot("$A$", A, dir(80));
dot("$B$", B, dir(225));
dot("$C$", C, dir(330));
dot("$A_1$", A1, dir(270));
dot("$B_1$", B1, dir(10));
dot("$C_1$", C1, dir(130));
dot("$D$", D, dir(175));
dot("$F$", F, dir(300));
dot("$L$", L, dir(90));
dot("$I_A$", Ia, dir(270));
dot("$I_B$", Ib, dir(45));
dot("$I_C$", Ic, dir(105));
dot("$M$", M, dir(115));
dot("$I$", I, dir(310));
dot("$T$", T, dir(225));
dot("$S$", S, dir(315));
dot("$I'$", I1, dir(180));
[/asy]
Claim: $(AB_1C_1)$ passes through $L$, the midpoint of arc $BAC$.

Proof. Let $L = (AB_1C_1) \cap (ABC)$. Then by spiral sim $\triangle LBC_1 \sim \triangle LCB_1$, but we know that $BC_1 = CB_1$, so in fact $\triangle LBC_1 \cong \triangle LCB_1$ and $LB = LC$ as desired. $\square$

Claim: If $(AB_1A_1C_1)$ meets $\overline{BC}$ again at $F$, then $L$, $I$, $F$ are collinear.

Proof. Let $S = \overline{AA_1} \cap (ABC)$ and $T = \overline{LF} \cap \overline{ABC}$. By Reim's $\overline{TS} \parallel \overline{BC}$. But since $\overline{AA_1}$ is isogonal to the $A$-mixtilinear chord, $T$ must be the $A$-mixtilinear intouch point and $I \in \overline{LFT}$ as desired. $\square$

Claim: $(AB_1A_1C_1)$ has diameter $\overline{AA_1}$.

Proof. Let $\triangle I_AI_BI_C$ be the excentral triangle and let $Be = \overline{I_BB_1} \cap \overline{I_CC_1}$ be the Bevan point (center of $(I_AI_BI_C)$). Then since $\measuredangle AB_1Be = \measuredangle AC_1Be = 90^\circ$, $Be$ lies on $(AB_1C_1)$. But by definition $Be$ also lies on $\overline{I_AA_1}$, so $A_1 = Be$, and it is the $A$-antipode on $(AB_1C_1)$. $\square$

This also implies $F$ is the foot of the altitude from $A$ to $\overline{BC}$.

Claim: $B_1$, $C_1$, $I$ are collinear.

Proof. Pappus on $\overline{I_CAI_B}$ and $\overline{BA_1C}$. $\square$

Claim: $\overline{AD}$ is tangent to $(AB_1A_1C_1)$.

Proof. Let $M$ be the midpoint of $\overline{AD}$. Note that $AIFD$ is cyclic with diameter $\overline{AD}$. The key idea is to notice that $(AIFD)$ is the $A$-Apollonian circle in $\triangle AC_1B_1$, which is true because both $\overline{AI}$ and $\overline{FI}$ are angle bisectors. Therefore $M$ is also the midpoint of $\overline{II'}$ where $I' = \overline{AL} \cap \overline{C_1B_1}$ and the result follows. $\square$

We are done because $MD^2 = MA^2 = MC_1 \cdot MB_1$. $\blacksquare$
Z K Y
The post below has been deleted. Click to close.
This post has been deleted. Click here to see post.
GeronimoStilton
1521 posts
#29
Y by
Ouch.

Solution with judicious helpings of help from awang11.

Let the arc midpoint of $\widehat{BAC}$ be $M$. If $A\ne M'=(AB_1C_1)\cap (ABC)$, then $M'B=M'C$ so config cases imply $M'=M$ as in IMO 2013/3.

Note that since $\triangle ABC$ is the orthic triangle of its excentral triangle $\triangle I_AI_BI_C$, the circumcenter $V$ of $\triangle I_AI_BI_C$ lies on $I_BB_1,I_CC_1,I_AA_1$. Then $V$ unconditionally lies on $(AB_1C_1)$ as $\angle VB_1A=\angle VC_1A=90^\circ$. Note $V,A_1,B_1$ must lie on both $(AB_1C_1)$ and $(CA_1B_1)$, but $V,A_1,C_1$ must lie on both $(AB_1C_1)$ and $(BC_1A_1)$ so the three circles cannot have a common chord and so $V=A_1$.

Next, note that by Pappus on $BA_1C$ and $I_CAI_B$, $I$ lies on $B_1C_1$.

Now, let $(AB_1C_1A_1)$ intersect $BC$ again at $E$ so $\angle AEA_1=90^\circ$. Then $E\in (AID)$.

Claim: $AD$ is tangent to $(AB_1C_1A_1E)$.

Solution: It is enough to check $\angle DIE=\angle DAE=\angle AME$, so we need to check $EIM$ collinear. Let $K$ denote the reflection of $I_A$ over $A_1$. Then by well-known geo configs and linearity, it is sufficient to show $IMK$ collinear. But this is clear: wrt $(I_AI_BI_C)$, $K$ is the $I_A$-antipode so standard results on the orthocenter configuration imply this. $\fbox{}$

Now it is sufficient to show $B_1C_1$ bisects $AD$ (which has midpoint $S$) by radical axis. Let the $I$-antipode wrt $(AD)$ be $I'$ so by rectangles $I'$ lies on $I_BI_C$. It would be enough to demonstrate that $I'$ lies on $B_1C_1$. Now angle chase: if $I''$ is $B_1C_1\cap I_BI_C$, then
\[\measuredangle B_1I''M = \measuredangle C_1B_1M+\measuredangle B_1MA=\measuredangle MC_1B_1+\measuredangle B_1C_1A=\measuredangle MC_1A, \]\[\measuredangle II'A=\measuredangle IEA = \measuredangle MEA=\measuredangle MC_1A.\]Thus $I''=I'$, done.
Z K Y
The post below has been deleted. Click to close.
This post has been deleted. Click here to see post.
jj_ca888
2726 posts
#31
Y by
Define $I_A, I_B, I_C$ excentral and $A'$ to be foot from $A$ to $BC$, and $M$ to be midpoint of arc $BAC$ in $(ABC)$.

We claim that $A_1$ is the bevan point $K$. Suppose otherwise, it is well known that $AB_1KC_1$ is cyclic with diameter $AK$. If $A_1$ is on $(AB_1C_1)$, then $\angle AA_1K = 90^{\circ}$ which means $A \in BC$, impossible. So $(AB_1C_1)$ has diameter $AA_1$. It follows that foot $A'$ lies on $(AB_1C_1)$. This circle is getting a lot of points so just call it $\omega$ from now on.

Next, note that by Pappus on triangles $I_BA_1I_C$ and $BAC$, we have that $B_1, I, C_1$ are collinear. Furthermore, note that $AIA'D$ cyclic with diameter $AD$. Next, we push to prove that $(AIA'D)$ and $\omega$ are orthogonal.

It is also well known that $M$ is center of spiral similarity sending $B_1C_1$ to $CB$, hence it turns out $M$ is also the midpoint of arc $B_1AC_1$ in $\omega$. We will show $A', I, M$ collinear, which would show by angle bisection that\[\frac{AB_1}{AC_1} = \frac{IB_1}{IC_1} = \frac{A'B_1}{A'C_1}\]and thus $AB_1A'C_1$ is harmonic. Let $AA_1$ and $MI$ hit $(ABC)$ again at $X$ and $T$. It is known that $T$ is the mixtillinear touchpoint, and that $AA_1, AT$ are isogonal. It follows that\[\angle MA'A = \angle MAA_1 = \angle MAX = \angle MTX\]but since $AT, AX$ isogonal it follows that $TX \parallel BC$ hence $MAA' = \angle MTX$ means that $A' \in \overline{MIT}$ as desired. It follows that $AIA'D$ is in fact the Appolonius circle to both $AB_1C_1$ and $A'B_1C_1$, and is indeed therefore orthogonal to $\omega$. It follows that center of $AIA'D$, the midpoint of $AD$ denoted as $O$, satisfies the property that $OA, OA'$ are both tangent to $\omega$ and thus $O \in \overline{B_1IC_1}$.

It follows that $OD^2 = OA^2 = OA'^2 = OB_1 \cdot OC_1$ so indeed $(DB_1C_1)$ is tangent to $AD$.
This post has been edited 1 time. Last edited by jj_ca888, Sep 3, 2021, 4:28 AM
Z K Y
The post below has been deleted. Click to close.
This post has been deleted. Click here to see post.
rafaello
1079 posts
#33
Y by
Define $M$ as the midpoint of arc $BC$, not containing $A$. Let $I_A,I_B,I_C$ be the excenters of $\triangle ABC$ wrt $A,B,C$ respectively. Let $K$ be the intersection of $AM$ and $BC$.

Note that the given condition of cyclic quadrilateral $AB_1A_1C_1$ yields that $A_1$ is the Bevan point of $\triangle ABC$.

Note that $\measuredangle DII_A=90^\circ=\measuredangle DA_1I_A$, we get that $DIA_1I_A$ is cyclic quadrilateral. Since $BICI_A$ is cyclic, we get that $$KD\cdot KA_1=KI\cdot KI_A=KB\cdot KC=KA\cdot KM,$$hence $A_1MDA$ is cyclic quadrilateral. Also, $I_AM\cdot I_AA$ is the power of $I_A$ wrt $(ABC)$ and because $I_AA_1$ is tangent to $(A_1B_1C)$ and $(ABC)$ is the nine-point circle of $\triangle I_AI_BI_C$, we get that $I_AM\cdot I_AA=I_AA_1^2$, hence $I_AA_1$ is tangent to $(A_1MDA)$. This means that $\measuredangle DAA_1=\measuredangle DA_1I_A=90^\circ$. Hence $DA$ is tangent to $(AB_1A_1C_1)$.

By Pappus theorem on triplets $I_C-A-I_b$ and $B-A_1-C$, we get that $I$ lies on $B_1C_1$. Now, easy angle chase implies $\measuredangle DAI=\measuredangle AIC_1\implies B_1C_1$ passes through the midpoint of $AD$. Hence, by PoP argument, we get that $AD$ is tangent to $(DB_1C_1)$.

[asy]import geometry;
size(10cm);defaultpen(fontsize(10pt));

pair O,A,B,C,I,A1,B1,C1,Ia,Ib,Ic,D,M,K; 
O=(0,0);A=dir(110);B=dir(193.3);C=dir(347.2);I=incenter(A,B,C);A1=2midpoint(B--C)-foot(I,B,C);
B1=2midpoint(A--C)-foot(I,A,C);C1=2midpoint(A--B)-foot(I,A,B);Ia=intersectionpoint(line(A,I),perpendicular(A1,line(A1,B)));
Ib=intersectionpoint(line(B,I),perpendicular(B1,line(B1,C)));
Ic=intersectionpoint(line(C,I),perpendicular(C1,line(C1,A)));
path w=circumcircle(A,B,C);path o=circumcircle(A1,B1,C1);D=intersectionpoint(perpendicular(I,line(A,I)),line(B,C));
M=intersectionpoints(w,A--Ia)[1];K=extension(B,C,A,I);

draw(A--B--C--cycle,red+1);draw(w,heavyblue+1);draw(o,cyan+1);draw(Ia--Ib--Ic--cycle,orange);
draw(Ia--A1,heavygreen);draw(Ib--A1,heavygreen);draw(Ic--A1,heavygreen);
draw(B--Ib,magenta);draw(A--Ia,magenta);draw(C--Ic,magenta);
draw(D--B,red+0.3);draw(A--D,red+0.3);draw(I--D,red+0.3);
draw(B1--C1,dashed+red+0.3);

dot("$A$",A,dir(A)); 
dot("$B$",B,dir(B)); 
dot("$C$",C,dir(C));
dot("$A_1$",A1,dir(A1)); 
dot("$B_1$",B1,dir(B1)); 
dot("$C_1$",C1,dir(C1));
dot("$I$",I,dir(I)); 
dot("$I_A$",Ia,dir(Ia)); 
dot("$I_B$",Ib,dir(Ib)); 
dot("$I_C$",Ic,dir(Ic));
dot("$D$",D,dir(D));
dot("$M$",M,dir(M)); 
dot("$K$",K,dir(K));
[/asy]
Z K Y
The post below has been deleted. Click to close.
This post has been deleted. Click here to see post.
Eyed
1065 posts
#34
Y by
Oranges are yummy

Let $N$ be the midpoint of major arc $\overarc{BAC}$ and $M$ the midpoint of minor arc $\overarc{BC}$. By spiral sim, $N$ also lies on $(AC_1 B_1 A_1)$. Now, observe by PoP, we have
\[CB_1 \left(\frac{CA}{CA_1} + \frac{BA}{BA_1}\right) = BC\]\[(s-a)\cdot \frac{s(b+c) - 2bc}{(s+b)(s-c)} = a \Rightarrow s^2 (b+c-a) - s(2bc) + abc = 0\]\[s \left(\frac{(b+c)^2 - a^2}{2} - 2bc\right) + abc = 0 \Rightarrow s(a-b+c)(a+b-c) = 2abc\]\[\frac{K^2}{s-a} = \frac{abc}{2} = K\cdot r_a \Rightarrow r_a = \frac{a}{\sin A} = 2R\]Therefore, $r_a = 2R$. Letting $I_A$ be the A-excenter, we have $I_a A_1 = MN$, so $A_1N || AI$ so $\angle ANA_1 = 90$. This means that $AA_1$ is the diameter of $(AB_1C_1)$, so the foot from $A$ to $BC$, which we denote as $X$, must also lie on this circle.

Let $T$ be the A-mixtilinear in-touch point. We first have $N,I,T$ collinear. Next,
\[\angle ANX = \angle AA_1 B = 180 - \angle BAA_1 -\angle B = 180 - \angle B - \angle TAC= 180 - \angle TAAN = \angle ANT\]Therefore, $N,X,T$ are collinear, so $N,I,X,T$ are collinear. Observe that $\angle AXD = \angle AID = 90$, so $(AIXD)$ is cyclic. We can also angle chase to get
\[\angle ADB = \angle AIN = 90 - \angle ANI = 90 - \angle AA_1 X = \angle XAA_1\]This means $AD\perp AA_1$, and since $AA_1$ is the diameter, we have $AD$ is tangent to $(AB_1C_1)$.

Now, observe that by pascal on $C_1B_1 A_1 XNA$, we have $B, C_1B_1\cap XN, A_1B_1\cap AN$ are collinear. However, $A_1B_1\perp AC$ (since $AA_1$ is diameter, and $AN$ is external bisector of $\angle BAC$, so $AN\cap A_1B_1 = I_B$, or the B-excenter. Since $I\in XN$, and $B,I,I_A$ collinear, this means $C_1, I, B_1$ are also collinear.

Finally, let $O$ be the midpoint of $AD$. If we show that $O,C_1,B_1$ are collinear, then we're done, since $O$ then $OA^2 = OC_1 OB_1 = OD\cdot OD'$, where $D'$ is the second intersection of $OD$ with $(DC_1B_1)$, but since $OD = OA$, we have $OD' = OA = OD$ so $D = D'$ and $OD$ is tangent to $(DC_1B_1)$. It remains to prove this collinearity. Since $O$ is the center of $(ADXI)$, we have
\[\angle OIA = 180 - \angle OAI - \angle AOI = 180 - \frac{1}{2}A - \angle AB_1C_1 - 2\angle AXN\]\[ = 180 - \frac{1}{2}A - \angle AB_1C_1 - \angle AC_1B_1 + \angle AB_1C_1 = 180 - \frac{1}{2}A - \angle AC_1B_1 = \angle C_1IA\]Therefore, $O, I, C_1$ are coillinear, so $O, C_1, B_1$ are collinear (since $I\in C_1B_1$). Therefore, $AD$ is tangent to $(DC_1B_1)$
This post has been edited 1 time. Last edited by Eyed, Oct 14, 2021, 4:45 AM
Reason: asdfa
Z K Y
The post below has been deleted. Click to close.
This post has been deleted. Click here to see post.
nprime06
187 posts
#35 • 1 Y
Y by Awesome_360
Solved with Awesome_360! Hopefully this relatively unique solution works :smile:

[asy]

size(12cm); defaultpen(fontsize(6pt)); dotfactor*=0.75;

pair A=dir(111.32); pair B=dir(192.94); pair C=dir(-13.14); pair I=incenter(A,B,C); pair P=foot(I,B,C); pair E=foot(I,A,C); pair F=foot(I,A,B); pair Q=extension(A,I,B,C); pair A_1=B+C-P; pair B_1=A+C-E; pair C_1=A+B-F; pair K=intersectionpoints(circumcircle(A,F,E),circumcircle(A,B,C))[1]; pair M=intersectionpoints((I--(10*Q-9*I)),unitcircle)[0];  pair D=extension(A,K,B,C); pair N=(A+D)/2; pair I_A=2*M-I; pair I_B=2*intersectionpoints((I--(10*(extension(B,I,A,C))-9*I)),unitcircle)[0]-I; pair I_C=2*intersectionpoints((I--(10*(extension(C,I,A,B))-9*I)),unitcircle)[0]-I; 

draw(10*A-9*B--10*B-9*A); draw(10*C-9*B--10*B-9*C); draw(10*A-9*C--10*C-9*A); draw(10*I_A-9*I_B--10*I_B-9*I_A); draw(10*I_C-9*I_B--10*I_B-9*I_C); draw(10*I_A-9*I_C--10*I_C-9*I_A); draw(unitcircle); draw(circumcircle(A,A_1,B_1),red); draw(circumcircle(A,F,E),darkgreen); draw(circumcircle(P,F,E)); draw(circumcircle(I,P,Q),linewidth(0.4)+gray); draw(circumcircle(foot(I_B,A,C),foot(I_B,B,C),foot(I_B,A,B)),linewidth(0.4)+orange); draw(circumcircle(foot(I_A,A,C),foot(I_A,B,C),foot(I_A,A,B)),linewidth(0.4)+orange); draw(circumcircle(foot(I_C,A,C),foot(I_C,B,C),foot(I_C,A,B)),linewidth(0.4)+orange); draw(circumcircle(I_A,I_B,I_C),blue); draw(circumcircle(B,I,C),blue); draw(circumcircle(D,C_1,B_1),red); draw(10*A-9*D--10*D-9*A,red); draw(10*B_1-9*C_1--10*C_1-9*B_1); draw(10*I-9*D--10*D-9*I,red); draw(B--K--C^^F--K--E,darkgreen); draw(A--A_1); draw(10*A-9*I_A--10*I_A-9*A^^10*B-9*I_B--10*I_B-9*B^^10*C-9*I_C--10*I_C-9*C^^10*A_1-9*I_A--10*I_A-9*A_1^^10*B_1-9*I_B--10*I_B-9*B_1^^10*C_1-9*I_C--10*I_C-9*C_1,linewidth(0.4)+gray); 

dot("$A$",A,dir(105)*3); dot("$B$",B,dir(210)*3); dot("$C$",C,dir(-20)*4); dot("$I$",I,dir(-90)*1.5); dot("$P$",P,dir(-100)*3); dot("$E$",E,dir(30)*1.5); dot("$F$",F,dir(F)*4.5); dot("$Q$",Q,dir(-80)*3.5); dot("$A_1$",A_1,dir(-90)*3); dot("$B_1$",B_1,dir(40)*2.5); dot("$C_1$",C_1,dir(-150)*4); dot("$K$",K,dir(90)*3); dot("$M$",M,dir(-90)*1.5); dot("$D$",D,dir(-80)*2); dot("$N$",N,dir(100)*1.5); dot("$I_A$",I_A,dir(I_A)*4); dot("$I_B$",I_B,dir(I_B)*3); dot("$I_C$",I_C,dir(I_C)*3); 

clip((2.7,2.2)--(2.7,-2.7)--(-3,-2.7)--(-3,2.2)--cycle);

[/asy]

Lemma. In any triangle $ABC$, a point $P$ has pedal triangle $DEF$. Then, the perpendiculars from $A$ to $EF$, $B$ to $DF$, and $C$ to $DE$ concur at $Q$, the isogonal conjugate of $P$. diagram
Proof. Assume WLOG that $C,P$ lie on the same side on the angle bisector of $\angle BAC$ (with the other case following similarly). Let $X=AQ\cap FE$ and $Y=AQ\cap FP$. Note that $\triangle YFX\sim\triangle YAF$, so \[\measuredangle QAF=\measuredangle EFP=\measuredangle EAP\]so $AQ,AP$ are isogonal. By symmetry, $BQ,BP$ are also isogonal and $CQ,CP$ are too.


Let $A_1'=I_BB_1\cap I_CC_1$. Note that since $ABC$ is the pedal triangle of the orthocenter in $\triangle I_AI_BI_C$, $I_AA_1'\perp BC$. As $A_1'$ also lies on $(AB_1C_1)$, it follows that $A_1=A_1'$. Let $K$ be the $A$-sharky devil point and $P$ the foot from $I$ to $BC$; $A,K,B$ are collinear by radical axis on $(AFE),(ABC),$ and $(BIC)$.


Claim. $\triangle KFE\sim\triangle AC_1B_1$.
Proof. First, we prove that $KT$ bisects $\angle BAC$; letting $M$ to be the midpoint of minor arc $BC$, it suffices to show that $K$ and $P$ swap under inversion in $(BIC)$. However, $A$ inverts to $Q=AI\cap BC$, so $K$ inverts to $(IQ)\cap BC=P$. Then, \[\frac{KF}{KE}=\frac{KB}{KC}=\frac{BP}{CP}=\frac{AC_1}{AB_1}\]by angle bisector theorem, and from $\measuredangle FKE=\measuredangle C_1AB_1$ the conclusion follows.


Then note that $B_1,I,C_1$ is collinear by Pascal on $A_1I_BBACI_C$. Let $N=B_1IC\cap AD$.


Claim. $NA=NI$.
Proof. Note that \[\measuredangle NAI=\measuredangle KAM=\measuredangle KBM=\measuredangle KBC+\measuredangle CBM=\measuredangle AC_1I+\measuredangle IAC_1=\measuredangle AIN.\]

Since $\measuredangle AID=90^\circ$, it follows that $N$ is the midpoint of $AD$. Since $\measuredangle AB_1C_1=\measuredangle KEF=\measuredangle KAF$, $ND$ is tangent to $(AA_1B_1C_1)$; thus, $ND^2=NA^2=NC_1\cdot NB_1$ which implies the conclusion.
This post has been edited 1 time. Last edited by nprime06, Oct 22, 2021, 5:48 AM
Z K Y
The post below has been deleted. Click to close.
This post has been deleted. Click here to see post.
MathLuis
1522 posts
#36 • 1 Y
Y by megarnie
Very nice one Ankan!
Let $I_A,I_B,I_C,I$ be the $A,B,C$ excenters and the incenter respectivily, let $O$ the center of $(I_AI_BI_C)$ and let the projection of $A$ to $BC$ be $H$ and let $I_BI_C \cap B_1C_1=G$
Claim 1: $H$ lies on $(AB_1A_1C_1)$.
Proof: Let $I_A,I_B,I_C$ be the $A,B,C$ excenters respectivily and let $O$ be the center of $(I_AI_BI_C)$, note that since $\angle BI_AC=90-\frac{\angle BAC}{2}$ we get $\angle I_BOI_C=180-\angle BAC$ hence $O$ lies on $(AB_1A_1C_1)$ but using orthic duality we have that $O-A_1-I_A$ hence $O \equiv A_1$ and since by more orthic duality we have $I_B-B_1-O$ and $I_C-C_1-O$ we get $\angle AB_1A_1=90=\angle AC_1A_1$ hence $AA_1$ is diameter of $(AB_1A_1C_1)$ as desired.
Claim 2: $I$ lies on $B_1C_1$
Proof: We use Pascal on the degenrate conic $\mathcal C$ passing through $A,B.C,I_B,I_C,A_1$ to get the desired result.
Claim 3: $AGDHI$ cyclic.
Proof: We do my favorite thing in geometry :love: (we must take in count Claim 2)
$$(G, I; B_1, C_1) \overset{A}{=} (I_BI_C \cap BC, AI \cap BC; B, C)=-1=(I_BI_C \cap BC, A; I_B, I_C)  \overset{A_1}{=} (H, A; B_1, C_1) \implies \frac{B_1H}{HC_1}=\frac{B_1A}{AC_1}=\frac{B_1I}{IC_1}=\frac{MB_1}{C_1M}$$Hence $HIAG$ is cyclic becuase its the apolonian circle of $\triangle AB_1C_1$ and since $\angle AHD=90$ we have $AGDHI$ cylcic wih diameters $GI$ and $AD$ (we let $K$ to be center of this circle)
Final steps: First letting $HI \cap (AB_1A_1C_1)=J \ne H$ we get by the previous ratios that $J$ is the midpoint of the arc $\widehat{B_1AC_1}$ hence we get $AI \parallel JA_1$ and now since $\angle ADH=\angle AIJ=\angle HJA_1=\angle HAA_1$ we get $AA_1$ tangent to $(AGDHI)$ (Claim 3 goes brrr) and becuase $\angle DAA_1=90$ we can get $KA$ tangent to $(AB_1A_1C_1)$ and now using tangent rule $KD^2=KA^2=KB_1 \cdot KC_1$ thus we are done :blush:
Z K Y
The post below has been deleted. Click to close.
This post has been deleted. Click here to see post.
CyclicISLscelesTrapezoid
372 posts
#37
Y by
I'm not usually a fan of config geo, but this is too good.

https://cdn.discordapp.com/attachments/872490629714808853/944709749323210762/unknown.png

Let $I_A$, $I_B$, and $I_C$ be the $A$, $B$, and $C$-excenters of $\triangle ABC$, respectively. Let $\Gamma$ and $O$ be the circumcircle and circumcenter of $\triangle ABC$, respectively. Let $\omega$ be the circumcircle of $AB_1A_1C_1$. Let $N$ be the midpoint of $\widehat{BAC}$ of $\omega$, let $K$ be the foot of the $A$-altitude, let $T$ be the $A$-mixtilinear intouch point of $\triangle ABC$, and let $M$ be the midpoint of $\overline{AD}$.

Claim 1 (USA EGMO TST 2019/5 + IMO 2013/3): $ANB_1A_1KC_1$ is cyclic.
Proof: Notice that since $NB=NC$, $NB_1=NC_1$, and $\measuredangle NBC_1=\measuredangle NCB_1$, $\triangle NBC_1 \cong \triangle NCB_1$. Thus, \[\measuredangle B_1AC_1=\measuredangle CNB=\measuredangle B_1NC_1,\]so $ANB_1A_1C_1$ is cyclic. Let $I'$ be the reflection of $I$ over $O$. Notice that lines $I_AA'$, $I_BB'$, and $I_CC'$ are concurrent. Also notice that $I'$ is the antipode of $A$ in $\omega$. Thus, either $\overline{AA_1} \parallel \overline{BC}$, or $A_1=I'$. The former is impossible, so $A_1$ is the antipode of $A$ in $\omega$. Since $\angle AKA_1=90^\circ$, $AB_1A_1KC$ is cyclic and the claim is proven.

Claim 2: $N$, $I$, $K$, and $T$ are collinear.
Proof: It's well-known that $N$, $I$, and $T$ are collinear. To prove that $N$, $K$, and $T$ are collinear, do a direct angle chase with $\measuredangle TAI=\measuredangle IAA_1$ to prove that $\measuredangle ANK=\measuredangle ANT$.

Claim 3: The circumcircle of $\triangle AKT$ is tangent to $\overline{AI}$.
Proof: Do a direct angle chase to prove that $\measuredangle ATI=\measuredangle IAK$.

Claim 4: $\measuredangle DAA_1=90^\circ$.
Proof: Do a direct angle chase with $\measuredangle AKI=\measuredangle IAT=\measuredangle A_1AI$.

By the previous claim, $\overline{AD}$ is tangent to $\omega$.

Claim 5: $I$, $B_1$, and $C_1$ are collinear.
Proof: Use Pappus on $\overline{BA_1C}$ and $\overline{I_BAI_C}$.

Claim 6: $\overline{B_1C_1} \parallel \overline{IM}$.
Proof: Do a direct angle chase to prove that $\measuredangle(\overline{AN},\overline{B_1C_1})=\measuredangle(\overline{AN},\overline{IM})$.

Hence, $I$, $B_1$, $C_1$, and $M$ are collinear. Therefore, we have \[MB_1 \cdot MC_1=MA^2=MD^2,\]so $\overline{AD}$ is tangent to the circumcircle of $\triangle DB_1C_1$.
This post has been edited 1 time. Last edited by CyclicISLscelesTrapezoid, Feb 19, 2022, 9:40 PM
Z K Y
The post below has been deleted. Click to close.
This post has been deleted. Click here to see post.
Inconsistent
1455 posts
#38 • 3 Y
Y by Mango247, Mango247, Mango247
Let $A_2 = (AA_1B_1C_1) \cap BC$. Then $a = BC = BA_2 + A_2C = \frac{(s-a)c}{s-c} + \frac{(s - a)b}{s-b}$.

Expanding gives $a^3 + a^2c + a^2b - ab^2 - 2abc - ac^2 - c^3 + c^2b + cb^2 - b^3 = 0$

I claim $A_2$ is the altitude from $A$. To confirm this, notice that expanding

$\frac{\frac{(s-a)c}{s-c}}{\frac{(s-a)b}{s-b}} = \frac{c\cos B}{b \cos C}$ using LoC yields the claim is equivalent to

$(b-c)(a^3 + a^2c + a^2b - ab^2 - 2abc - ac^2 - c^3 + c^2b + cb^2 - b^3) = 0$ as desired.

Now, notice that using barycentric coordinates

$\text{det} \begin{vmatrix}
a & b & c\\
s-a & s-b & 0\\
s-a & 0 & s-c
\end{vmatrix} = 0 \Longleftrightarrow a^3 + a^2c + a^2b - ab^2 - 2abc - ac^2 - c^3 + c^2b + cb^2 - b^3 = 0$ so $B_1IC_1$ is collinear.

Thus by angle bisector theorem $\frac{s - b}{s - c} = \frac{AC_1}{AB_1} = \frac{IC_1}{IB_1}$. Then notice by the lemma on my blog that if $T = AA_2 \cap (ABC)$ and $M$ is the midpoint of arc $BAC$ then $\triangle B_1C_1A_2$ is spiral similar to $\triangle CBT$ through $M$ so $\frac{A_2C_1}{A_2B_1} = \frac{TB}{TC} =  \frac{s-b}{s-c}$ by ratio lemma in $\triangle ABC$. Notice $(AIA_2D)$ is concyclic and thus an Appolonius circle of $C_1B_1$. Hence $\frac{DB_1}{DC_1} = \frac{s - c}{s - b}$.

Now notice that computing the $D$ through barycentric coordinates, that $A_2, D$ are harmonic conjugates wrt $B, C$ via cross ratio, so if $D_1 = B_1C_1 \cap AD$ then $\frac{B_1D_1}{C_1D_1} = \frac{s-c}{s-b} \cdot \frac{s-c}{s-b}$ by ratio lemma in $\triangle AB_1C_1$ so it follows that since $\left( \frac{D_1B_1}{D_1C_1} \right)^2 = \frac{DB_1}{DC_1}$, we have $AD = D_1D$ is tangent to $(B_1C_1D)$ as desired.

EDIT:

WOW are you kidding me? I did bary because it was way too simple but I came here hoping to see some nice synthetic solution given all the nice observations made in the solution... but it was just a Bevan point configuration knowledge check? SMH.
This post has been edited 2 times. Last edited by Inconsistent, Nov 12, 2022, 10:53 PM
Reason: edit
Z K Y
The post below has been deleted. Click to close.
This post has been deleted. Click here to see post.
asdf334
7585 posts
#39
Y by
nooooooooooo.


Angle chase to get that $AA_1$ is a diameter, so draw $A_1B_1\cap AB=V$ and $A_1C_1\cap AC=U$ and note $A_1$ is orthocenter of $\triangle AUV$. Let $K=AA_1\cap B_1C_1$ and let $Z$ be the harmonic conjugate of $B_1C_1$ with respect to $K$. So now $Z$ lies on $UV$ and $BC$. Let $E$ be the foot of the altitude from $A$ to $BC$, note $AB_1EC_1$ harmonic. Now we need to show that $I$ lies on $B_1C_1$. This is simple by Menelaus bash: let $AI\cap BC=J$ and Menelaus on $\triangle CAJ$ to show $ZB_1I$ collinear.

Now $(AEID)$ is obviously an Apollonian circle. From here we can let $O$ be the midpoint of $AD$, notice that $(AEID)$ and $(B_1C_1D)$ orthogonal and we're done.


rip bevan point

oh i never drew the excenters that might have been the issue LOL!
This post has been edited 2 times. Last edited by asdf334, Oct 21, 2023, 7:50 PM
Z K Y
The post below has been deleted. Click to close.
This post has been deleted. Click here to see post.
Ywgh1
139 posts
#40
Y by
USA TST 2019

Nice problem.
Here is a sketch of the solution.

Firstly we show that $B_1IC_1$ are collinear by pappus theorem.
Let $(AB_1A_1C_1) \cap BC=H’$ then we can easily how that $H’$ is the $A$ feet of altitude.
The main idea is showing that $AH’B_1C_1$ is harmonic.
While showing that the tangents at $A$ and $H’$ intersect at $M$ which the intersection of $AD$ with $B_1C_1$.
We how that $M$ is the Center of $AIH’D$ hence by power of point we get that.
$$MD^2=MA^2=MB_1 \cdot MC_1$$Hence we are done.
This post has been edited 2 times. Last edited by Ywgh1, Aug 18, 2024, 6:45 PM
Z K Y
The post below has been deleted. Click to close.
This post has been deleted. Click here to see post.
mcmp
53 posts
#41
Y by
Let’s AK this. Parts of this solution will be similar to other solutions above.

Step 1: Proving as much as possible without the sticky condition.
We prove the usual claim that $(AB_1C_1)$ is tangent to $\overline{AD}$, and $\overline{IM}\parallel\overline{B_1C_1T}$ where $M$ is the midpoint of $AD$.
[asy]
 /* Geogebra to Asymptote conversion, documentation at artofproblemsolving.com/Wiki go to User:Azjps/geogebra */
import graph; size(10cm); 
real labelscalefactor = 0.5; /* changes label-to-point distance */
pen dps = linewidth(0.7) + fontsize(10) + blue; defaultpen(dps); /* default pen style */ 
pen dotstyle = blue; /* point style */ 
real xmin = -21, xmax = 18, ymin = -11, ymax = 11;  /* image dimensions */
pen qqwuqq = rgb(0,0.39215686274509803,0); pen ccqqqq = rgb(0.8,0,0); pen ffvvqq = rgb(1,0.3333333333333333,0); 

draw((-5.00130207163223,8.65950215591446)--(-9.152407121756008,-4.029074816583814)--(9.101968524336803,-4.141759165135286)--cycle, linewidth(0.5) + ccqqqq); 
 /* draw figures */
draw(circle((0,0), 10), linewidth(0.5) + qqwuqq); 
draw((-5.00130207163223,8.65950215591446)--(-9.152407121756008,-4.029074816583814), linewidth(0.5) + ccqqqq); 
draw((-9.152407121756008,-4.029074816583814)--(9.101968524336803,-4.141759165135286), linewidth(0.5) + ccqqqq); 
draw((9.101968524336803,-4.141759165135286)--(-5.00130207163223,8.65950215591446), linewidth(0.5) + ccqqqq); 
draw((-19.759779416499267,-3.9635954564539646)--(-2.845029399842456,0.5141498919484042), linewidth(0.5) + ffvvqq); 
draw((-19.759779416499267,-3.9635954564539646)--(-9.152407121756008,-4.029074816583814), linewidth(0.5) + ffvvqq); 
draw((-19.759779416499267,-3.9635954564539646)--(-5.00130207163223,8.65950215591446), linewidth(0.5) + ffvvqq); 
draw(circle((-3.9231657357373417,4.586826023931432), 4.212964376160934), linewidth(0.5) + qqwuqq); 
draw(circle((-1.0781363358948877,4.072676131983027), 6.035743729142097), linewidth(0.5) + qqwuqq); 
draw(circle((-0.061728871928146724,-9.999809475503547), 10.876125385867832), linewidth(0.5) + qqwuqq); 
draw(circle((-3.1536888609567755,-2.271612354266112), 6.2508635466551326), linewidth(0.5) + qqwuqq); 
draw(circle((5.97349896208963,-2.3279545285418446), 3.616242395309652), linewidth(0.5) + qqwuqq); 
draw((-11.056367069694842,3.4805345203159774)--(3.8660813226853814,0.6107526625138946), linewidth(0.5) + ffvvqq); 
draw((-6.953736677062792,2.6915450756578996)--(2.8228754214215614,-4.10299829324825), linewidth(0.5) + ffvvqq); 
draw((2.8228754214215614,-4.10299829324825)--(3.8660813226853814,0.6107526625138946), linewidth(0.5) + ffvvqq); 
draw((-5.00130207163223,8.65950215591446)--(-2.845029399842456,0.5141498919484042), linewidth(0.5) + ffvvqq); 
draw((-7.77946021700683,6.283311128060428)--(-7.199972516325446,1.9388822636727459), linewidth(0.5) + ffvvqq); 
draw((-7.199972516325446,1.9388822636727459)--(0.23458513001919146,3.906990328265279), linewidth(0.5) + ffvvqq); 
draw((0.23458513001919146,3.906990328265279)--(-7.77946021700683,6.283311128060428), linewidth(0.5) + ffvvqq); 
draw((-7.77946021700683,6.283311128060428)--(-2.845029399842456,0.5141498919484042), linewidth(0.5) + ffvvqq); 
 /* dots and labels */
dot((0,0),dotstyle); 
label("$O$", (0.12817675840733747,0.2926007430266706), NE * labelscalefactor); 
dot((-5.00130207163223,8.65950215591446),dotstyle); 
label("$A$", (-4.89166511339295,8.939495990405241), NE * labelscalefactor); 
dot((-9.152407121756008,-4.029074816583814),dotstyle); 
label("$B$", (-9.041014175054459,-3.7406826106163535), NE * labelscalefactor); 
dot((9.101968524336803,-4.141759165135286),dotstyle); 
label("$C$", (9.210318410855255,-3.856748318634858), NE * labelscalefactor); 
dot((-2.845029399842456,0.5141498919484042),dotstyle); 
label("$I$", (-2.7154330880460043,0.81489642910994), NE * labelscalefactor); 
dot((2.8228754214215614,-4.10299829324825),dotstyle); 
label("$A_1$", (2.9427701778560533,-3.798715464625606), NE * labelscalefactor); 
dot((3.8660813226853814,0.6107526625138946),dotstyle); 
label("$B_1$", (3.987361550022587,0.9019457101238182), NE * labelscalefactor); 
dot((-6.953736677062792,2.6915450756578996),dotstyle); 
label("$C_1$", (-6.835765722702887,2.9911284544568955), NE * labelscalefactor); 
dot((-19.759779416499267,-3.9635954564539646),dotstyle); 
label("$D$", (-19.573977177733674,-3.6826497566071015), NE * labelscalefactor); 
dot((-7.199972516325446,1.9388822636727459),dotstyle); 
label("$C_2$", (-7.09691356574452,2.2367013523366177), NE * labelscalefactor); 
dot((0.23458513001919146,3.906990328265279),dotstyle); 
label("$B_2$", (0.360308174444345,4.209818388651191), NE * labelscalefactor); 
dot((-2.873314018840767,-4.067835688470851),dotstyle); 
label("$A_2$", (-2.74444951505063,-3.76969903762098), NE * labelscalefactor); 
dot((-7.77946021700683,6.283311128060428),dotstyle); 
label("$S$", (-7.677242105837039,6.560148976025903), NE * labelscalefactor); 
dot((-0.06172887192814658,-9.999809475503545),dotstyle); 
label("$M_a$", (0.04112747739345968,-9.718066573569326), NE * labelscalefactor); 
dot((-11.056367069694842,3.4805345203159774),dotstyle); 
label("$T$", (-10.927081930355143,3.7745719835817995), NE * labelscalefactor); 
dot((2.845029399842456,-0.5141498919484042),dotstyle); 
label("$V$", (2.971786604860679,-0.2296949430565987), NE * labelscalefactor); 
clip((xmin,ymin)--(xmin,ymax)--(xmax,ymax)--(xmax,ymin)--cycle); 
 /* end of picture */
[/asy]
I claim that if $S$ is the $A$-sharkydevil point then $\triangle SB_2C_2\stackrel{+}{\sim}\triangle AB_1C_1$. We however have angles $\measuredangle C_2SB_2=\measuredangle C_2AB_2=\measuredangle C_1AB_1$, and ratios $\frac{SC_2}{SB_2}=\frac{BC_2}{CB_2}=\frac{AC_1}{AB_1}$, as desired. So $\measuredangle DAC_1=\measuredangle SAC_2=\measuredangle SB_2C_2=\measuredangle AB_1C_1$, as desired.

Now further notice that $\measuredangle (\overline{B_1C_1},\overline{B_2C_2})=\measuredangle (\overline{AC_1},\overline{SC_2})=\measuredangle AC_2S=\measuredangle AIS=\measuredangle IDA=\measuredangle MID=\measuredangle(\overline{MI},\overline{B_2C_2})$, as desired.

Step 2: Interpreting the condition
Introduce the Bevan point $V$; we claim that $V=A_1$. However if $AA_1B_1C_1$ cyclic and $V\neq A_1$ then as $AV$ is the diameter of $(AA_1B_1C_1)$ then $\overline{AA_1}\perp\overline{A_1V}$. But then again $\overline{A_1V}\perp\overline{BC}$ by definition of the Bevan point, and hence $\overline{AA_1}\parallel\overline{BC}$, which is a contradiction.

So we have that $V=A_1$. Now $I_aA_1=r_a$, the $A$-exradius, whilst $I_aV=2R$ due to the excentre-orthocentre duality so $r_a=2R$, where $I_a$ is the $A$-excentre.

Since I'm actually very dumb, let's draw in $M_b$ and $M_c$ being defined as per the diagram below:
[asy]
 /* Geogebra to Asymptote conversion, documentation at artofproblemsolving.com/Wiki go to User:Azjps/geogebra */
import graph; size(10cm); 
real labelscalefactor = 0.5; /* changes label-to-point distance */
pen dps = linewidth(0.7) + fontsize(10) + blue; defaultpen(dps); /* default pen style */ 
pen dotstyle = blue; /* point style */ 
real xmin = -21.65110060692449, xmax = 20.977842173753757, ymin = -11.055305559931867, ymax = 12.390612969441175;  /* image dimensions */
pen qqwuqq = rgb(0,0.39215686274509803,0); pen ccqqqq = rgb(0.8,0,0); pen ffvvqq = rgb(1,0.3333333333333333,0); 

draw((-5.476119010842156,8.367324577132951)--(-9.78267371561353,-2.0734741314624983)--(9.786194275204696,-2.056794012036894)--cycle, linewidth(0.5) + ccqqqq); 
 /* draw figures */
draw(circle((0,0), 10), linewidth(0.5) + qqwuqq); 
draw((-5.476119010842156,8.367324577132951)--(-9.78267371561353,-2.0734741314624983), linewidth(0.5) + ccqqqq); 
draw((-9.78267371561353,-2.0734741314624983)--(9.786194275204696,-2.056794012036894), linewidth(0.5) + ccqqqq); 
draw((9.786194275204696,-2.056794012036894)--(-5.476119010842156,8.367324577132951), linewidth(0.5) + ccqqqq); 
draw((-3.595934733432582,2.0708437955383086)--(-3.5924066956343808,-2.0681976692155213), linewidth(0.5) + ffvvqq); 
draw((-5.476119010842156,8.367324577132951)--(-3.595934733432582,2.0708437955383086), linewidth(0.5) + ffvvqq); 
draw((-9.244471768878217,3.813101324960256)--(9.786194275204696,-2.056794012036894), linewidth(0.5) + ffvvqq); 
draw((-9.78267371561353,-2.0734741314624983)--(5.640013234593206,8.257738837818346), linewidth(0.5) + ffvvqq); 
draw((5.640013234593206,8.257738837818346)--(5.486618265075023,-8.411225757322281), linewidth(0.5) + ffvvqq); 
draw((-9.244471768878217,3.813101324960256)--(5.486618265075023,-8.411225757322281), linewidth(0.5) + ffvvqq); 
draw((-9.244471768878217,3.813101324960256)--(-5.476119010842156,8.367324577132951), linewidth(0.5) + ffvvqq); 
draw((-5.476119010842156,8.367324577132951)--(5.486618265075023,-8.411225757322281), linewidth(0.5) + ffvvqq); 
draw((-7.836531405897288,2.6447476274567396)--(3.595927255225547,-2.0620704742838716), linewidth(0.5) + ffvvqq); 
draw((3.595927255225547,-2.0620704742838716)--(5.57158428578019,0.8217731824280499), linewidth(0.5) + ffvvqq); 
 /* dots and labels */
dot((0,0),dotstyle); 
label("$O$", (0.12962484507830163,0.33461508928060524), NE * labelscalefactor); 
dot((-5.476119010842156,8.367324577132951),dotstyle); 
label("$A$", (-5.3322084486960994,8.693884337679233), NE * labelscalefactor); 
dot((-9.78267371561353,-2.0734741314624983),dotstyle); 
label("$B$", (-9.661710449858735,-1.7302243266584978), NE * labelscalefactor); 
dot((9.786194275204696,-2.056794012036894),dotstyle); 
label("$C$", (9.920960140015337,-1.7302243266584978), NE * labelscalefactor); 
dot((-3.595934733432582,2.0708437955383086),dotstyle); 
label("$I$", (-3.4671922020414256,2.3994545052197083), NE * labelscalefactor); 
dot((3.595927255225547,-2.0620704742838716),dotstyle); 
label("$A_1=V$", (3.726441892198029,-1.7302243266584978), NE * labelscalefactor); 
dot((5.57158428578019,0.8217731824280499),dotstyle); 
label("$B_1$", (5.691369723494917,1.1672116279657274), NE * labelscalefactor); 
dot((-7.836531405897288,2.6447476274567396),dotstyle); 
label("$C_1$", (-7.696782618561846,2.9656201515255916), NE * labelscalefactor); 
dot((-3.5924066956343808,-2.0681976692155213),dotstyle); 
label("$A_2$", (-3.4671922020414256,-1.7302243266584978), NE * labelscalefactor); 
dot((0.008523800852425989,-9.999996367240291),dotstyle); 
label("$M_a$", (0.12962484507830163,-9.656543374940862), NE * labelscalefactor); 
dot((5.640013234593206,8.257738837818346),dotstyle); 
label("$M_b$", (5.757977446589726,8.593972753037018), NE * labelscalefactor); 
dot((-9.244471768878217,3.813101324960256),dotstyle); 
label("$M_c$", (-9.095544803552851,4.131255305684762), NE * labelscalefactor); 
dot((5.486618265075023,-8.411225757322281),dotstyle); 
label("$U$", (5.624762000400107,-8.091261882212832), NE * labelscalefactor); 
clip((xmin,ymin)--(xmin,ymax)--(xmax,ymax)--(xmax,ymin)--cycle); 
 /* end of picture */
[/asy]
We are going to force an application of Pascal to get $B_1$, $I$, $C_1$ collinear. To do that I claim the hexagon $AM_cBUCM_b$ works where $U$ is the antipode of $A$ on $(ABC)$. All I actually need to show is that $B_1=\overline{AC}\cap\overline{M_bU}$. I will do this using the ratio lemma. Let $B_1'=\overline{AC}\cap\overline{M_bU}$; then:
\begin{align*}
\frac{AB_1}{B_1C}&=\frac{AM_b}{M_bC}\frac{AU}{UC}\\
		 &=\frac{1}{\cos(\angle B)}\\
		 &=\frac{2ac}{a^2+c^2-b^2}
\end{align*}I claim that with the condition $2R=r_a$, $\frac{2ac}{a^2+c^2-b^2}=\frac{AB_1}{B_1C}=\frac{s-c}{s-a}$. Notice however $2R=\frac{abc}{2rs}=\frac{s}{s-a}r$, so $2(rs)^2=abc(s-a)$. Now $\frac{2ac}{a^2+2ac+c^2-b^2}=\frac{2ac}{(a+c-b)(a+b+c)}=\frac{ac}{2s(s-b)}$, whilst $\frac{s-c}{2s-a-c}=\frac{s-c}{b}$, so it suffices to show that $abc=2s(s-b)(s-c)$, i.e. $\frac{2(rs)^2}{s-a}=2s(s-b)(s-c)$, or $(rs)^2=s(s-a)(s-b)(s-c)$, which is just Heron’s formula squared. So the Pascal works.

Part 3: The finish
[asy]
 /* Geogebra to Asymptote conversion, documentation at artofproblemsolving.com/Wiki go to User:Azjps/geogebra */
import graph; size(10cm); 
real labelscalefactor = 0.5; /* changes label-to-point distance */
pen dps = linewidth(0.7) + fontsize(10) + blue; defaultpen(dps); /* default pen style */ 
pen dotstyle = blue; /* point style */ 
real xmin = -23.259339754245026, xmax = 19.541271812197504, ymin = -10.447898799466639, ymax = 13.092437562076874;  /* image dimensions */
pen qqwuqq = rgb(0,0.39215686274509803,0); pen ccqqqq = rgb(0.8,0,0); pen ffvvqq = rgb(1,0.3333333333333333,0); 

draw((-5.529490044046149,8.332150973955915)--(-9.78267371561353,-2.0734741314624983)--(9.786194275204696,-2.056794012036894)--cycle, linewidth(0.5) + ccqqqq); 
 /* draw figures */
draw(circle((0,0), 10), linewidth(0.5) + qqwuqq); 
draw((-5.529490044046149,8.332150973955915)--(-9.78267371561353,-2.0734741314624983), linewidth(0.5) + ccqqqq); 
draw((-9.78267371561353,-2.0734741314624983)--(9.786194275204696,-2.056794012036894), linewidth(0.5) + ccqqqq); 
draw((9.786194275204696,-2.056794012036894)--(-5.529490044046149,8.332150973955915), linewidth(0.5) + ccqqqq); 
draw((-17.336151248875133,-2.0799125676251085)--(-3.6344941786712726,2.0592624641139796), linewidth(0.5) + ffvvqq); 
draw((-17.336151248875133,-2.0799125676251085)--(-9.78267371561353,-2.0734741314624983), linewidth(0.5) + ffvvqq); 
draw((-17.336151248875133,-2.0799125676251085)--(-5.529490044046149,8.332150973955915), linewidth(0.5) + ffvvqq); 
draw(circle((-4.58199211135871,5.1957067190349475), 3.2764363104863987), linewidth(0.5) + qqwuqq); 
draw(circle((-0.9474979326874364,3.1364442549209683), 6.927482949728449), linewidth(0.5) + qqwuqq); 
draw(circle((0.008523800852427224,-9.999996367240291), 12.597511800380618), linewidth(0.5) + qqwuqq); 
draw(circle((-3.074089768471129,-2.0663682977880526), 6.708587710444645), linewidth(0.5) + qqwuqq); 
draw(circle((6.710344226938108,-2.058028238075522), 3.0758502958916925), linewidth(0.5) + qqwuqq); 
draw((-11.432001519352662,3.12684157533726)--(5.574190950974938,0.8002948325823711), linewidth(0.5) + ffvvqq); 
draw((-7.857008451664908,2.6377615833571366)--(3.634496544141483,-2.0620375985781543), linewidth(0.5) + ffvvqq); 
draw((3.634496544141483,-2.0620375985781543)--(5.574190950974938,0.8002948325823711), linewidth(0.5) + ffvvqq); 
draw((-5.529490044046149,8.332150973955915)--(-3.6344941786712726,2.0592624641139796), linewidth(0.5) + ffvvqq); 
draw((-7.575344228330761,6.527952192096369)--(-7.455155307994771,3.62091525913628), linewidth(0.5) + ffvvqq); 
draw((-7.455155307994771,3.62091525913628)--(-1.3174867198163902,5.47506212933665), linewidth(0.5) + ffvvqq); 
draw((-1.3174867198163902,5.47506212933665)--(-7.575344228330761,6.527952192096369), linewidth(0.5) + ffvvqq); 
draw((-7.575344228330761,6.527952192096369)--(-3.6344941786712726,2.0592624641139796), linewidth(0.5) + ffvvqq); 
draw((-5.5257089064128975,4.203787559149821)--(-3.6309759845503167,-2.0682305449212386), linewidth(0.5) + ffvvqq); 
draw((-3.6344941786712726,2.0592624641139796)--(0.008523800852428653,-9.999996367240293), linewidth(0.5) + ffvvqq); 
draw((-7.575344228330761,6.527952192096369)--(0.008523800852428653,-9.999996367240293), linewidth(0.5) + ffvvqq); 
draw((-3.6344941786712726,2.0592624641139796)--(-7.455155307994771,3.62091525913628), linewidth(0.5) + ffvvqq); 
draw((-3.6344941786712726,2.0592624641139796)--(-1.3174867198163902,5.47506212933665), linewidth(0.5) + ffvvqq); 
draw((-3.6344941786712726,2.0592624641139796)--(3.634496544141483,-2.0620375985781543), linewidth(0.5) + ffvvqq); 
 /* dots and labels */
dot((0,0),dotstyle); 
label("$O$", (0.1472446961532296,0.3191300477166152), NE * labelscalefactor); 
dot((-5.529490044046149,8.332150973955915),dotstyle); 
label("$A$", (-5.403459616369786,8.678624494287465), NE * labelscalefactor); 
dot((-9.78267371561353,-2.0734741314624983),dotstyle); 
label("$B$", (-9.650082795227755,-1.7540245750329557), NE * labelscalefactor); 
dot((9.786194275204696,-2.056794012036894),dotstyle); 
label("$C$", (9.911134209747932,-1.7205865972466723), NE * labelscalefactor); 
dot((-3.6344941786712726,2.0592624641139796),dotstyle); 
label("$I$", (-3.497494882551642,2.392284670466186), NE * labelscalefactor); 
dot((3.634496544141483,-2.0620375985781543),dotstyle); 
label("$A_1=V$", (3.758546297071818,-1.7205865972466723), NE * labelscalefactor); 
dot((5.574190950974938,0.8002948325823711),dotstyle); 
label("$B_1$", (5.697949008676245,1.1216415145874168), NE * labelscalefactor); 
dot((-7.857008451664908,2.6377615833571366),dotstyle); 
label("$C_1$", (-7.710680083623329,2.960730292833004), NE * labelscalefactor); 
dot((-17.336151248875133,-2.0799125676251085),dotstyle); 
label("$D$", (-17.207065774927766,-1.7540245750329557), NE * labelscalefactor); 
dot((-7.455155307994771,3.62091525913628),dotstyle); 
label("$C_2$", (-7.30942435018793,3.9638696264215056), NE * labelscalefactor); 
dot((-1.3174867198163902,5.47506212933665),dotstyle); 
label("$B_2$", (-1.1902744152980995,5.802958404667093), NE * labelscalefactor); 
dot((-3.6309759845503167,-2.0682305449212386),dotstyle); 
label("$A_2$", (-3.497494882551642,-1.7205865972466723), NE * labelscalefactor); 
dot((-7.575344228330761,6.527952192096369),dotstyle); 
label("$S$", (-7.443176261333063,6.872973693828162), NE * labelscalefactor); 
dot((0.008523800852428653,-9.999996367240293),dotstyle); 
label("$M_a$", (0.1472446961532296,-9.678825310382122), NE * labelscalefactor); 
dot((-11.432001519352662,3.12684157533726),dotstyle); 
label("$T$", (-11.288543706755634,3.462299959627255), NE * labelscalefactor); 
dot((3.6344941786712726,-2.0592624641139796),dotstyle); 
dot((-5.5257089064128975,4.203787559149821),dotstyle); 
label("$P$", (-5.403459616369786,4.532315248788324), NE * labelscalefactor); 
clip((xmin,ymin)--(xmin,ymax)--(xmax,ymax)--(xmax,ymin)--cycle); 
 /* end of picture */
[/asy]
So what happens now is that $\overline{IM}$ is the same as $\overline{C_1B_1}$, so $DM^2=AM^2=MC_1\cdot MB_1$, thus done by PoP!
Z K Y
The post below has been deleted. Click to close.
This post has been deleted. Click here to see post.
bin_sherlo
716 posts
#43
Y by
Let $I,O$ be the incenter and circumcenter of $ABC$, $W$ be the circumcenter of $(AB_1C_1)$. Let $I'$ be the reflection of $I$ over $O$ (Bevan point). Let $N$ be the midpoint of arc $BAC$.

First, we will prove two lemmas which hold with/without the condition that $A_1\in (AB_1C_1)$.

Lemma: $A,N,B_1,C_1,I'$ are concyclic.
Proof: Since $IA\perp AN$ and $O$ lies on the perpendicular bisector of $AN$, we get $I'N\perp AN$. Also $I'B_1\perp AC$ and $I'C_1\perp AB$ hence $A,B_1,C_1,N,I'$ lie on the circle with diameter $AI'$.

Lemma: $I'A\perp AD$.
Proof: Let $S$ be $A-$sharky devil. Since $IS\perp \overline{ASD}$ and $O$ lies on the perpendicular bisector of $AS$, we see that $I'A\perp \overline{ASD}$.

Since $A_1\in (AB_1C_1)$ and $AI'$ is the diameter, if $I'\neq A_1$ then $\measuredangle AA_1I'=90=\measuredangle BA_1I'$ however this would yield $A\in BC$ which is not true. Thus, $I'=A_1$. Now, we will proceed by assuming the condition holds.

Claim: If $AH$ is the altitude, then $N,I,H$ are collinear.
Proof: Note that $\measuredangle AHA_1=90$ hence $H\in (AB_1C_1NA_1)$. Let $T$ be $A-$mixtilinear touchpoint and $A'$ be the antipode of $A$ on $(ABC)$. Since $\measuredangle A_1NA=90=\measuredangle A'NA$, we observe the collinearity of $A',A_1,N$. Also $H,A',A_1,T$ are concyclic which can be seen under $\sqrt{bc}$ inversion. We have $\measuredangle HTA'=\measuredangle HA_1N=180-\measuredangle NAH=\measuredangle NTA'$ thus, $N,H,T$ are collinear and since $N,I,T$ are collinear we get the result.

Claim: $I$ lies on $B_1C_1$.
Proof: Pascal at $B_1C_1ANHA_1$ gives it.

Claim: $DI$ bisects $\measuredangle B_1DC_1$.
Proof: We will utilise that $AD$ is tangent to $(AB_1C_1)$. Let $B_1C_1$ meet $AD,AN$ at $L,K$. Since $\measuredangle LIA=\frac{\measuredangle A}{2}+\measuredangle C_1AL=\measuredangle AIL$ thus, $LI=LA$ which implies also $LA=LD$. Since $\measuredangle IAK=90$ and $LI=LA$, we also get $LA=LK$. So $KDIA$ is a rectangle. $-1=(AN,AI;AB,AC)=(K,I;C_1,B_1)$ and since $\measuredangle KAI=90$, we must have $\measuredangle C_1DI=\measuredangle IDB_1$.

The Finish: $AD$ is tangent to $(DB_1C_1)$.
Proof:
\[\measuredangle LDC_1=\measuredangle LDI-\measuredangle C_1DI=\measuredangle DIL-\measuredangle IDB_1=\measuredangle DB_1C_1\]As desired.$\blacksquare$
Z K Y
N Quick Reply
G
H
=
a